Sei sulla pagina 1di 108

ed co

A Little Help to Get Started


Apps

For All Previous Year AIIMS PG Questions Visit

www.medicoapps.org/aiimspg/
Anatomy
1. Finger with two dorsal interossei attached?

A. Little

B. Index

C. Middle

D. Ring

Ans.C. Middle finger

 1st: largest; lateral side of index finger


 2nd and 3rd: both attach to either side of 3rd (middle) finger
 4th: medial side of 4th (ring) finger

2. Arrange the following in sequence from Lateral to medial

A. Inferior colliculus
B. Medial geniculate body
C. Cochlear nucleus
D. Lateral lemniscus

Answer: c->d->a->b

Auditory pathway: SC-SLIM-41,42

For All Previous Year AIIMS PG Questions Visit

www.medicoapps.org/aiimspg/
 S- Spiral ganglia (cochlea)
 C- Cochlear nuclei (ponto-medullary junction)
 S-Superior olivary nucleus (pons)
 L-Lateral lemniscus (brain stem)
 I-Inferior colliculus (mid-brain)
 M-Medial geniculate body (meta-thalamus)
 41,42 – Brodmann area (cerebral cortex)

3. Which is not a branch of the external carotid artery supplying nasal septum?

A. Sphenopalatine
B. greater palatine
C. superior labial
D. anterior ethmoidal

Ans.D.anterior ethmoidal

 ed co
The nasal septum also derives its blood supply from :

labial artery, and the angular artery.


Apps
Branches from the external carotid artery are the sphenopalatine artery, the greater palatine artery, the superior

The main branches from the interior carotid are the anterior ethmoidal artery, and the posterior ethmoidal artery
that supplies the septum, and these derive from the ophthalmic artery.
A Little Help to Get Started

4. Mark true or false among the following:


Content 's of Carotid Sheath are
A. Internal jugular vein
i)true
ii)false
B. Cervical sympathetic trunk
i)true
ii)false
C.Vagus nerve
i)true

For All Previous Year AIIMS PG Questions Visit

www.medicoapps.org/aiimspg/
ii)false
D. Internal carotid artery
i)true
ii)false
E.Deep cervical lymph nodes
i)true
ii)false
Ans.A. i) B. ii) C.i) D. i) E. i)

The carotid sheath also is a tubular fascial investment that extends superiorly between the cranial base
and inferiorly to the root of the neck. The carotid sheath contains the common and internal carotid
arteries, internal jugular vein, and vagus nerve [cranial nerve (CN) X]. In addition, the carotid sheath
contains deep cervical lymph nodes, sympathetic fibers, and the carotid sinus nerve. The cervical
sympathetic trunk lies behind the sheath but is not included within it.

Common carotid artery divides at the level of?

ed co
5.

A.

B.
Hyoid bone
Cricoid cartilage
Apps
C. Superior border of thyroid cartilage
A Inferior
L i tborder
D.
t l ofethyroidHcartilage
elp to Get Started
Ans. C. Superior border of thyroid cartilage
Common carotid artery divides into internal and external branches at the level of fourth cervical vertebra
which also corresponds to the upper border of thyroid cartilage.
Branches of External carotid artery are:
 Superior thyroid artery
 Lingual artery
 Facial artery
 Ascending pharyngeal artery
 Occipital artery
 Posterior auricular artery
 Maxillary artery
 Superficial temporal artery

For All Previous Year AIIMS PG Questions Visit

www.medicoapps.org/aiimspg/
Branches of Internal carotid artery:

 Cervical portion has no branches

Branches from other portions are:

 Tympanic branch
 Artery of the pterygoid canal
 Cavernous branches
 Hypophyseal branches
 Anterior meningeal branch
 Ophthalmic artery
 Anterior cerebral artery
 Middle cerebral artery
 Posterior communicating artery
 Anterior choroidal artery

ed co
6. True about levator Ani except
A. Converge downwards & medially
B. Supports viscera
Apps
C. Made up of iliococcygeus& pubococcygeus
AD. Attached
L i t tot pelvic
l ebrim H e l p t o G e t S t a r t e d
Ans. d. Attached to pelvic brim
The levator ani is made up of three parts:
 Iliococcygeus muscle
 Pubococcygeus muscle
 Puborectalis muscle
ORIGIN & Insertion:
 The levator ani arises, in front, from the posterior surface of the superior pubic ramus lateral to the
symphysis; behind, from the inner surface of the spine of the ischium; and between these two
points, from the obturator fascia.
 The fibers pass downward and backward to the middle line of the floor of the pelvis; the most
posterior are inserted into the side of the last two segments of the coccyx; those placed more
anteriorly unite with the muscle of the opposite side, in a median fibrous ridge called the
anococcygeal body or raphe, which extends between the coccyx and the margin of the anus.

Action:Supports the viscera in pelvic cavity

For All Previous Year AIIMS PG Questions Visit

www.medicoapps.org/aiimspg/
7. Couniad classified liver into 8 segments,which differentiates the segments anatomically

A. Portal vein
B. Hepatic vein
C. Bile duct
D. Hepatic artery

Ans. B) Hepatic vein

 The Couinaud classification divides the liver into 8 functional segments.


 The hepatic veins are found at the periphery of each segment, whereas the center has branches of
the portal veins, hepatic arteries, and bile ducts.
 The middle hepatic vein divides the liver into left and right lobes. The left hepatic vein divides the
left lobe into lateral (2, 3) and medial (4a, 4b) segments. The right hepatic vein divides the right
lobe into anterior (5, 8) and posterior (6, 7) segments. The caudate lobe (1) has hepatic veins that
often drain directly into the IVC.

8. What is the action of muscle attached at the point marked in the below image?

For All Previous Year AIIMS PG Questions Visit

www.medicoapps.org/aiimspg/
A. Flexion
B. Extension
C. Adduction
D. Abduction

Ans. d) Abduction

 Attachments of Gluteus Medius: Origin & Insertion. Outer surface of ilium, between the posterior
and anterior gluteal lines. Lateral and superior surfaces of the greater trochanter of femur
 The function of the gluteus medius muscle is to work with other muscles on the side of the hip to
help pull the thigh out to the side in a motion called hip abduction.

9.Assertion - Distal pole of scaphoid mc goes to avn after scaphoid fracture


Reason- Blood supply of scaphoid is from distal to proximal

A. Assertion is true but reason is false


B. Both assertion reason are true and reason explains assertion
C. Both assertion and reason ar false
D. Reason is true but assertion is false

Ans. D. Reason is true but assertion is false


 The main blood supply to the scaphoid enters through the non-articular dorsal ridge at the waist of
the bone and the volar tubercle at the distal aspect of the bone.
 A dorsal branch of the radial artery accounts for 80% of the blood supply of the scaphoid.

For All Previous Year AIIMS PG Questions Visit

www.medicoapps.org/aiimspg/
 A separate volar arterial branch to the scaphoid enters the tubercle and accounts for 20–30% of the
scaphoid’s blood supply, mainly to the distal portion.
 The proximal pole of the scaphoid relies entirely on intramedullary blood flow.
 The unusual retrograde nature of the scaphoid’s blood supply renders it especially prone to non-
union and proximal pole avascular necrosis

10. Identify the Anatomical structures of heart coronal section

A. Papillary muscle
B. Cusps of valves
C. Left ventricle
D. Membranous IVseptum

Ans. A. ii) Cusps of valves


B. i) Papillary muscle
C. iv) Membranous IVseptum
D. iii) Left ventricle

For All Previous Year AIIMS PG Questions Visit

www.medicoapps.org/aiimspg/
11. Muscle not supplied by the nerve passing through Foramen marked in the image below?

A. Superior Rectus
B. Inferior Rectus
C. Medial Rectus
D. Superior Oblique

Ans. D. Superior oblique muscle

The trochlear nerve supplies only the superior oblique muscle.

For All Previous Year AIIMS PG Questions Visit

www.medicoapps.org/aiimspg/
12. Golgi tendon organs used to detect ?
A.Dynamic
B. Static
C. Tension of muscle
D. All

Ans. C. Tension of muscle

The Golgi tendon organ (GTO) (also called Golgi organ, tendon organ, neurotendinous organ or
neurotendinous spindle) is a proprioceptive sensory receptor organ that senses changes in muscle tension.
It lies at the origins and insertion of skeletal muscle fibers into the tendons of skeletal muscle.

13. False about innervation of parotid gland:

ed co
A. Postganglionic parasympathetic fibre secretomotor
B. Preganglionic parasympathetic fibre relay in Otic ganglion
C. Preganglionic parasympathetic nerve begin in inferior petrosal nucleus
Apps
D. Sympathetic nerve are vasomotor
A Little Help to Get Started
Ans. C. Preganglionic parasympathetic nerve begin in inferior petrosal nucleus
Nerve supply:

 PARASYMPATHETIC:auriculo temporal nerve


 SYMPATHETIC SUPPLY- plexus around the external carotid artery.
 SENSORY NERVES: auriculotemporal nerve, except for parotid fascia & overlying skin which are
innervated by Great auricular nerve (C2, C3).

14. Identify the surface marking

For All Previous Year AIIMS PG Questions Visit

www.medicoapps.org/aiimspg/
A. Thoracic duct
B. Right bronchopulmonary segt
C. Right atrium border
D. Aorta

Ans. C. Right atrium border

15. Arrange the following layers of epidermis from surface to deep layer

A. Corneocytes
B. Merkel cells
C. Melanocytes
D. Langerhans

Ans.A ->D-> C -> B

The epidermis is composed of 4 or 5 layers, depending on the region of skin being considered.Those layers
in descending order are:

For All Previous Year AIIMS PG Questions Visit

www.medicoapps.org/aiimspg/
 Cornified layer (stratum corneum)
o Composed of 10 to 30 layers of polyhedral, anucleated corneocytes (final step of
keratinocyte differentiation), with the palms and soles having the most layers.
 Clear/translucent layer (stratum lucidum, only in palms and soles)
o This narrow layer is found only on the palms and soles. The epidermis of these two areas is
known as "thick skin" because with this extra layer, the skin has 5 epidermal layers instead
of 4
 Granular layer (stratum granulosum)
o Keratinocytes lose their nuclei and their cytoplasm appears granular. Lipids, contained into
those keratinocytes within lamellar bodies, are released into the extracellular space through
exocytosis to form a lipid barrier. Those polar lipids are then converted into non-polar lipids
and arranged parallel to the cell surface.
 Spinous layer (stratum spinosum)
o Keratinocytes become connected through desmosomes and start produce lamellar bodies,
from within the Golgi, enriched in polar lipids, glycosphingolipids, free sterols, phospholipids
and catabolic enzymes.
o Langerhans cells, immunologically active cells, are located in the middle of this layer.
 Basal/germinal layer (stratum basale/germinativum).
o Composed mainly of proliferating and non-proliferating keratinocytes, attached to the
basement membrane by hemidesmosomes.
o Melanocytes are present, connected to numerous keratinocytes in this and other strata

ed co
through dendrites.
o
such as the fingertips and lips.
Apps
Merkel cells are also found in the stratum basale with large numbers in touch-sensitive sites

A Little Help to Get Started

For All Previous Year AIIMS PG Questions Visit

www.medicoapps.org/aiimspg/
Physiology

16. Anion gap is normally calculated by using which of the following methods?
A. [K+] + [HCO3 + Cr]
B. [Na + [a- + HCO3-]
C. [Na+ - [HCO3-+ Cl-]
D. [Mg] + [HCO3- + Cl-]

Ans. C. [Na+ - [HCO3-+ Cl-]


A L responsible
ed co
Formula for calculating anion gap = Na+ -( HCO3 -+ Cl-)
Normal anion gap is (in mEq/L) = 10-12 mEq/L

i t t l efor rhythm
17. What is H egeneration
l p oft inspiration?
o Get Started
Apps

A. Dorsal nucleus of vagus


B. Pre botzinger complex
C. ventral respiratory neurons
D. Pneumotaxiccenter

Ans. B. Pre botzinger complex


[Ref: Ganong 25Ie p656, 24Ie p658]
 Rhythmic respiration is initiated by a small group of synaptically coupled pacemaker cells in the pre-
Botzinger complex (pre-BOTC) on either side of the medulla between the nucleus ambiguus and the
lateral reticular nucleus.
 The main components of the respiratory control pattern generator responsible for automatic
respiration are located in the medulla.
 Rhythmic respiration is initiated by a small group of synaptically coupled pacemaker cells in the pre-
Botzinger complex (pre-BOTC) on either side of the medulla between the nucleus ambiguus and the
lateral reticular nucleus.

For All Previous Year AIIMS PG Questions Visit

www.medicoapps.org/aiimspg/
18. In female adrenal gland secretes which hormone?
A. Progesterone
B. Testosterone
C. Estrogen
D. DHEA
Ans. D. DHEA
 DHEA is an endogenous steroid hormone. This means it is naturally made by the body, and it spurs
specific tissues or cells into action.
o It is also known as androstenolone, 3β-hydroxyandrost-5-en-17-one and 5-androsten-3β-ol-
17-one.
 DHEA is one of the most abundant steroid hormones in the human body. It is produced by the
adrenal glands, the gonads, and the brain.
o It is normally found in the form of dehydroepiandrosterone sulfate (DHEAS).
 The body holds DHEAS in reserve and converts it to specific hormones when needed.
o It is important for creating estrogen and androgen sex hormones and contributes to the

ed co
development of so-called androgenic effects, or masculinization.

Apps
o These changes include the production of oilier skin, changes in body odor, and the growth of
armpit and pubic hair.

A Little Help to Get Started

19. What is the sequence of clotting factors in coagulation pathway?


A. XII, XIII, X, XI
B. XIII, XII, XI, X
C. XII, XI, IX, X
D. X, XIII, XII, XI

Ans. C. XII, XI, IX, X

For All Previous Year AIIMS PG Questions Visit

www.medicoapps.org/aiimspg/
20. Which of the following is not a monomeric intermediate filament?
A. Vimentin
B. Keratin
C. Desmin
D. Tubulin

Ans. D. Tubulin

 The building block of a microtubule is the tubulin subunit, a heterodimer of α- and β-tubulin.

21. Which of the following does golgi tendon organ detect?


A. Muscle Tension
B. Dynamic muscle length
C. Static muscle length
D. Muscle action

For All Previous Year AIIMS PG Questions Visit

www.medicoapps.org/aiimspg/
Ans. A. Muscle Tension

(Ref.: Ganong 25/e p232; Guyton 13/e p697, 701).

 Golgi tendon organ senses muscle tension.


 The Golgi organ (also called Golgi tendon organ, GTO, tendon organ, neurotendinous organ or
neurotendinous spindle) senses changes in muscle tension.
 It is a proprioceptive sensory receptor organ that is at the origin and insertion of skeletal muscle
fibers into the tendons of skeletal muscle.

 It provides the sensory component of the Golgi tendon reflex.

22. Which hormone act by crossing cell membrane

A. Thyroxine
B. Insulin
C. Glucagon
D. Calcitonin
Ans. A. Thyroxine


ed co
for DNA and function as ligand (hormone)-regulated transcription factors.
Apps
Intracellular receptors are transcription factors that have binding sites for the hormone (ligand) and

Steroid hormones and the steroid derivative vitamin D3 fulfill this requirement.
A Little Help to Get Started
 Thyroid hormones must be actively transported into the cell.

23. Which of the following hormones is controlled by feedforward control?


A. ADH
B. Insulin
C. Corticosteroids
D. Prolactin

Ans. A. ADH
 Osmolarity and volume status are the two greatest factors that affect ADH secretion.
 However, a variety of other factors promote ADH secretion as well.
 These include angiotensin II, pain, nausea, hypoglycemia, nicotine, opiates, and certain
medications.
 ADH secretion is also negatively affected by ethanol, alpha-adrenergic agonists, and atrial
natriuretic peptide.

For All Previous Year AIIMS PG Questions Visit

www.medicoapps.org/aiimspg/
 Ethanol’s inhibitory effect helps to explain the increased diuresis experienced during intoxicated
states as well as increased free water loss; without appropriate ADH secretion, more water is
excreted by the kidneys.

24. Anatomical dead space measured by


A. Bohler's method
B. Xenon dilution technique
C. Spirometry
D. Single breath nitrogen test

Ans. D. Single breath nitrogen test


(Ref: Ganong 25/e p633, 634, 24/e p633, 634)
Anatomical dead space – Calculation:
 By Bohr's equation - Uses single breath nitrogen inhalation technique.
Xenon/Helium dilution technique:

ed co
 Used to measure functional residual capacity of lung.
Spirometry: Apps
 Cannot measure residual or dead space volumes.

A Little Help to Get Started

25. Which are all the factors determining the GFR?


A. Afferent arteriolar resistance
B. Efferent arteriolar resistance
C. Arterial pressure
D. All of the above

Ans. D. All of the above

Determinants of GFR:

 Increase glomerular capillary hydrostatic pressure increases GFR. Glomerular hydrostatic pressure
is determined by the following variables :i) Arterial pressure, ii)Afferent arteriolar resistance; iii)
Efferent arteriolar resistance.

For All Previous Year AIIMS PG Questions Visit

www.medicoapps.org/aiimspg/
 Contraction of mesangial cells (therefore decrease in GFR) is produced by : - Endothelin,
angiotensin II, vasopressin, norepinephrine, PAF, PDGF, thromboxane A2,PGF2, Leukotrienes C4 &
D4, Histamine.

26. Which of the following nucleus has cardio inhibitory function?

A. Nucleus Ambiguus
B. Nucleus Tractus Solitarius
C. Rostral Ventrolateral Medulla
D. Dorsal motor nucleus of vagus

Ans. A. Nucleus Ambiguus


 The nucleus ambiguus in its "external formation" contains cholinergic preganglionic
parasympathetic neurons for the heart.
 These neurons are cardioinhibitory.
This cardioinhibitory effect is one of the means by which quick changes in blood pressure are

ed co

achieved by the central nervous system (the primary means being changes in sympathetic nervous

Apps
system activity, which constricts arterioles and makes the heart pump faster and harder).

A Little Help to Get Started

Biochemistry

27. If changes
made in some of the sequence of amino acids in the chain of human proinsulin,
which of the following changes will not make any difference in the biochemical activity of the
molecule?

For All Previous Year AIIMS PG Questions Visit

www.medicoapps.org/aiimspg/
A. Change in A1-A4

B. Change in B29,B30

C. Intra disulphide bonds

D. Change in A5 and A6

Answer-a-change in A1-A4
 A6-A11 is intra disulphide bonds.A7-B7,A20-B19 are interchain bonds
 So A1-A4 is the only amino acid not involved in insulin formation, so the best Answer is change in
A1-A4

28. intermediate of kreb cycle used in heme synthesis;


A. succinyl-CoA
B. Alpha ketoglutarate
C. Citrate
D. Aspartate
Answer-2- (A) succinyl-CoA
 All the carbon and nitrogen atoms of the porphyrin molecule are provided by glycine
 (a nonessential amino acid) and succinyl coenzyme A (an intermediate in the citric acid cycle) that
condense to form ALA in a reactioncatalyzed by ALA synthase(ALAS)
 Heme synthesis also requires a functional tricarboxylic acid cycle and an oxygen supply.
 Heme synthesis starts in mitochondria with the condensation of succinyl-CoA with the amino acid
glycine, activated by pyridoxal phosphate.
 ALA synthase is the rate-limiting enzyme of heme synthesis. ALA molecules enter the cytoplasm,
where their union in the presence of ALA dehydratase yields porphobilinogen(PBG) and water
molecules.
For All Previous Year AIIMS PG Questions Visit

www.medicoapps.org/aiimspg/
29. Mother to children's transmission is a key feature of which pattern of inheritance?
A. Codominance
B. Autosomal dominant inheritance
C. Recessive inheritance
D. Mitochondrial inheritance

Ans-d-Mitocondrialinheritance , it is the only non-chromosomal DNA in human cells.

 Mitochondria! DNA, is always maternally inherited.


 Mitochondrial and nuclear DNA are located in different places in the cell. During fertilization, the
sperm and egg cell nuclei fuse to form an embryo.
 The egg cell is very large compared to the sperm, so although the cells nuclei fuse, the rest of the
cell mass in the embryo comes from the egg only.
 Nuclear DNA is therefore co-inherited but the mitochondrial DNA, which is located outside of the
nucleus, is always maternally inherited because all mitochondria in a foetus and later adult are
derived from the mitochondria in the mother 's egg.

ed co
 So, in diseases showing mitochondria! inheritance all children from affected mother will inherit the

Apps
disease but it will not be transmitted from an affected father to his children.

A young
30. A
L iman
t twasl onehighH elp to Get Started
protein diet and raw eggs. After 3 days he developed weakness.
Blood investigation revealed hypoglycemia. Hypoglycemia is due to inhibition of which of the
following enzymes

A. Glucose 6 phosphatase
B. Glycogen phosphorylase
C. Pyruvate Carboxylase
D. Glucokinase

Ans- c- Pyruvate Carboxylase


 aminoacids in raw eggs are more easily accessed by the body. When aminoacids enter the
bloodstream, they stimulate the hormone insulin, which is an anabolic hormone that drives them
into the cells.
 Raw egg contain avidin which is antagonist of biotin, so this person is deficient in biotin,
 Carboxylase reaction need biotin, so answer is pyruvate carboxylase.

For All Previous Year AIIMS PG Questions Visit

www.medicoapps.org/aiimspg/
31. Sickle cell anemia is a
A. Gene deletion
B. Gene modification
C. Point mutation
D. Frame shift mutation
Answer- c- point mutation.
 Sickle-cell anemia is caused by a point mutation in the β-globin chain of hemoglobin, causing the
hydrophilic amino acid glutamic acid to be replaced with the hydrophobic amino acid valine at the
sixth position.
 It is caused by a point mutation (Base substitution mutation) at the sixth position of the β-globin
chain leading to subsitution of a valine residue for a glutamic acid residue resulting in sickle
hemoglobin (HbS).
 Sickle cell anemia is an autosomal recessive disorder.

A. DNA repair
ed co
32. what does not occur in

B. DNA replication
AC. RNA
L iediting
5'→ 3'direction?

ttle Help to Get Started


Apps
D. Transcription

Ans -C- RNA editing


 In rna editing editosome can edit only in 3→ 5 direcƟon, along with primary rna transcript.
 DNA replication goes in the 5' to 3' direction because DNA polymerase acts on the 3'-OH of the
existing strand for adding free nucleotides.
 In transcription the complementary RNA is created in the opposite direction, in the 5' → 3'
direction, matching the sequence of the sense strand with the exception of switching uracil for
thymine.
 When the strand containing the mis-match is identified, an endonuclease nicks the strand and the
mis-matched nucleotide(s) is/are removed by an exonuclease.

33. Hepcidin decreases iron absorption by inhibition of


A. Hephaestin

For All Previous Year AIIMS PG Questions Visit

www.medicoapps.org/aiimspg/
B. Ferroportin
C. Divalent metal ion transporter
D. Transferrin
Ans - b). Ferroportin
 By inhibiting ferroportin, hepcidin prevents enterocytes from allowing iron into the hepatic portal
system, thereby reducing dietary iron absorption.
 The iron release from macrophages is also reduced by ferroportin inhibition.

34.Thiamine deficiency is best diagnosed by


A. Thiamine level in blood
B. Transketolase level in blood
C. Aldolase level in blood
D. Thiamine level in urine

Ans- b- Transketolase level in blood

ed co
 Pyruvate dehydrogenase in carbohydrate metabolism, which catalyzes the conversion of pyruvate


to acetyl CoA.
Apps
α-Ketoglutarate dehydrogenase in citric acid cycle, which catalyzes the conversion of a-ketogluta
rate to succinyl CoA.
 Branched-chain keto acid dehydrogenase which catalyzes the oxidative decarboxylation of
A Little Help to Get Started
branched chain keto amino acids, i.e. leucine, isoleucine and valine.
 Thiamine diphosphate is also the coenzyme for transketolase, in the pentose phosphate pathway.
Therefore, thiamine nutritional status is best assessed by erythrocyte (preferred) or whole blood
transketolase activity.

34. Kcat/km is a measure of -


A. Enzyme efficiency
B. Speed of enzymatic reaction
C. Concentration of substrate
D. Enzyme turn over

Answer-Enzyme efficiency

 "The Km of an enzyme is the concentration of the substrate that enables the enzyme to

For All Previous Year AIIMS PG Questions Visit

www.medicoapps.org/aiimspg/
 Function at half maximum activity and is therefore a measure of the specificity of a substrate for
the enzyme" .
 Actually enzyme specificity is not measured by alone.
 It is measured by the ratio Kcat /Km which is a second order rate constant for the reaction
between substrate and free enzyme.
 This ratio is important, for it provides a direct measure of enzyme eficiency and specificity.
 Note: Km, is turnover number and measures tJre rate of the catalytic process

35. Enzyme involved in gluconeogenesis are all except:


A. Pyruvate carboxylase
B. PEP carboxykinase
C. Pyruvate kinase
D. Glucose-6-phosphatase

Answer-c- Pyruvate kinase

ed co
 Mitochondrial pyruvate carboxylase catalyzes the carboxylation of Pyruvate to Oxaloacetate, It


is an ATP-requiring reaction, Biotin is the coenzyme.
Apps
Phosphoenolpyruvate Carboxykinase: Catalyzes the decarboxylation and phosphorylation of
oxaloacetate to phosphoenolpyruvate(PEPCK) (Cytosol) using GTP as the phosphate donor.
The conversion of glucose-6-phosphate to glucose is catalyzed by glucose 6-phosphatase
A L i t t l e H e l p t o G e t S t a r t e d

36. Which of the following leads to an increase in enzyme activity -


A. Increase in temperature
B. Decrease in activation energy
C. Extremes of pH value
D. Low substrate concentration

Answer-B. -Decrease in activation energy

 The enzymes speed up chemical reactions by lowering the magnitude of the activation energy
banier, i.e.,free energy of Activation

For All Previous Year AIIMS PG Questions Visit

www.medicoapps.org/aiimspg/
37. The letters A through E designate certain regions on the titration curve for glycine (shown
below). Which one of the following statements concerning this curve is correct?

A. Represents the region where glycine is deprotonated


B. Point B represents a region of minimal buffering
C. Point C represents the region where the net charge on glycine is zero
D. Point D represents the pK of glycine’s carboxyl group
E. Point E represents the pI for glycine

Answer-C. Point C represents the region where the net charge on glycine is zero
 C represents the isoelectric point or pI, and as such is midway between pK1
 and pK2 for this monoamino monocarboxylic acid.
 Glycine is fully protonated at Point A. Point B represents a region of maximum buffering, as does
Point D.
 Point E represents the region where glycine is fully deprotonated

38. Assertion : Central dogma is the flow of information from DNA to mRNA and then decoding
the information present in mRNA in the form of protein.

Reason : In retroviruses, reverse of central dogma occurs.


A. If both the assertion and the reason are true and the reason is a correct explanation of the
assertion
B. If both assertion and reason are true but the reason is not the correct explanation of the assertion
C. If the assertion is true but the reason is false
D. If both the assertion and reason are false
E. If the assertion is false but reason is true

For All Previous Year AIIMS PG Questions Visit

www.medicoapps.org/aiimspg/
Answer: BIf both assertion and reason are true but the reason is not the correct explanation of the
assertion
 Biosynthesis of protein is under direct control of DNA in most cases or else under the control of
genetic RNA where DNA is absent. Sequences of bases in a particular segment of a polynucleotide
chain will determine the sequence of amino acids in a particular polypeptide.
 The relationship is popularly known as central dogma. Flow of information is one way i.e., from
DNA, information is transferred to RNA (mRNA) and from RNA to protein.
 Temin (1970) reported that retroviruses operate a central dogma reverse or teminism inside host
cells. Genomic RNA of these viruses first synthesizes DNA through reverse transcription. DNA then
transfers information to messenger RNA which takes part in translation of the coded information to
form polypeptide.

(i) One way flow of information (central dogma)

DNA(transcription) → RNA (translaVon) → proteins

(i) Reverse flow of transcription information

DNA(transcription) ↔RNA(translation) → proteins

Microbiology

39. Correct

AB. Iodine
L i →t GenVon
ed co
order of gram staining is –
A. Gention violet → IodinE → Carbol fuchsin
t l evioletH e lfuchsin
→ Carbol p to Get Started
Apps
C. Carbol fuchsin → Iodine → Gention violet
D. Carbol fuchsin → GenVon violet → Iodine

Answer-A- Gention violet → IodinE → Carbol fuchsin

1. Application of the primary stain (crystal violet).Gention violet also known as crystal violet stains
all cells blue/purple
2. Application of mordant: The iodine solution (mordant) is added to form a crystal violet-iodine (CV-
I) complex; all cells continue to appear blue.
3. Decolorization step: The decolorization step distinguishes gram-positive from gram-negative cells.
The organic solvent such as acetone or ethanol, extracts the blue dye complex from the lipid-rich,
thin-walled gram-negative bacteria to a greater degree than from the lipid-poor, thick-walled,
gram-positive bacteria. The gram-negative bacteria appear colorless and gram-positive bacteria
remain blue.
4. Application of counterstain (safranin): The red dye safranin stains the decolorized gram-negative
cells red/pink; the gram-positive bacteria remain blue.

For All Previous Year AIIMS PG Questions Visit

www.medicoapps.org/aiimspg/
NOTE →If you are struggling to remember the staining reagents used in this procedure and their order you
can remember this sentence “Come InAnd Stain” i.e. the order is Crystal violet, Iodine, Alcohol/Acetone
and the final one is Safranin.

40. Antibody in chronic allergy ?

A. IgM
B. IgA
C. IgG
D. IgE

Answer-2- D- IgE
 When someone has allergies, their immune system makes an antibody called immunoglobulin E
(IgE). These antibodies respond to allergens. The symptoms that result are an allergic reaction.
 IgE also has an essential role in type I hypersensitivity, which manifests in various allergic diseases,
such as allergic asthma, most types of sinusitis, allergic rhinitis, food allergies, and specific types of
chronic urticaria and atopic dermatitis.

41. identify the given image and transmission associated with image?

A. Malaria
B. Filariasis
C. Dengue
D. A And C

For All Previous Year AIIMS PG Questions Visit

www.medicoapps.org/aiimspg/
Answer- B-i.e., Filariasis , the given image is of culex mosquito.

Mosquito species Disease transmitted


Anopheles Malaria, filariasis (not in India), arboviruses of febrile and encephalitic disease
Culex Bancroftian filariasis, Japanese encephalitis, West-Nile fever, St.
Louis encephalitis, western equine encephalitis
Aedes yellow fever (in Africa), Dengue, Dengue haemorrhagic fever, Chickengunya,
Chickengunya haemorrhagic fever, Rift valley fever, Sindbis, Bancroftian filariasis
(not in India)
Mansonia Brugian filariasis, Bancroftian filariasis, Chikungunya

42. Chancre redux is a clinical feature of

A. Early relapsing syphilis


B. Late syphilis
C. Chancroid
D. Recurrent herpes simplex infection

Ans. is 'a' i.e., Early relapsing syphilis

 Chancre redux is the appearance of relapsing lesion at the site of the healed lesion, it is due to
relapse of original infection (not due to reinfection)

43. Infectious cause of erythema multiforme in given image is

A. Staphylococcus
B. TB

For All Previous Year AIIMS PG Questions Visit

www.medicoapps.org/aiimspg/
C. HSV
D. EBV

Answer-C-Herpes simplex virus


 Herpes simplex is the primary cause of erythema multiforme, and the virus is present in 70 percent
of recurrent erythema multiforme cases. Both types of herpes simplex virus (HSV) can cause the
condition, but HSV-1, which also causes cold sores, is responsible for most cases.

Causes of Erythema multiforme


 Idiopathic → Most common cause
 Viral → HSV (most important) HBV, Mumps, Adenovirus
 Bacteria → Streptococci, tuberculosis
 Fungal → Coccidioidomycosis, Histoplasmosis.
 Drugs →AnVbioVcs (Sulphonamide), Phenytoin, NSAIDS.
 Autoimmune disease → SLE, thyroidiVs, RA
 Others → Sarcoidosis, Pregnancy, Malignancy.

ed co
44. clostridium difficile diarrhoea associated with:

A. Aminopenicillins Apps
B. Carbapenems

AC. Macrolide
Little Help to Get Started
D. Fluoroquinolones
Answer-D-fluoroquinolones,
 Clostridium difficile colitis results from a disturbance of the normal bacterial flora of the colon,
colonization by C difficile, and the release of toxins that cause mucosal inflammation and damage.
 It is a spore-forming bacillus that is responsible for the development of antibiotic-associated
diarrhea and colitis.
 The antibiotics that most often lead to C. difficile infections include fluoroquinolones,
cephalosporins,penicillins and clindamycin. Once established, C. difficile can produce toxins that
attack the lining of the intestine

45. The dimorphic fungus exists in two phases: a unicellular yeast form at 37 C and a mycelium at
25 C; the fungus is:

A. Malassezia furfur
B. Cryptococcus neoformans

For All Previous Year AIIMS PG Questions Visit

www.medicoapps.org/aiimspg/
C. Aspergillus
D. Histoplasma capsulatum

Answer-d- Histoplasma capsulatum


 Dimorphic fungi are fungi that can exist in the form of both mold and yeast. An example is
Penicillium marneffei, a human pathogen that grows as a mold at room temperature, and as a yeast
at human body temperature.

Mnemonics: Body Heat Probably (Changes) Shape


 Blastomyces dermatitidis, Histoplasma capsulatum, Paracoccidioides brasiliensis, (Coccidioides
immitis) is in parentheses because it changes to a spherule of endospores, not yeast, in the heat),
Sporothrixschenckii.

46. identify the disease caused by the given organism?

A. Giardia
B. Leishmania
C. Trypanosoma
D. Plasmodium

Answer-b-Leishmania
 It is a disease caused by protozoan parasites of the genus Leishmania and spread by the bite of
certain types of sandflies.
 The disease can present in three main ways: cutaneous, mucocutaneous, or visceral
leishmaniasis(Kala azar/Black fever).
 The cutaneous form presents with skin ulcers, while the mucocutaneous form presents with ulcers
of the skin, mouth, and nose, and the visceral form starts with skin ulcers and then later presents
with fever, low red blood cells, and enlarged spleen and liver.
 Microscopic examination:direct visualization of the intracellular Amastigotes (Leishman-Donovan
bodies).Amastigotes( formed after the macrophage phagocytizes an infective promastigote) are
seen within blood and spleen monocytes or, less commonly, in circulating neutrophils and in
For All Previous Year AIIMS PG Questions Visit

www.medicoapps.org/aiimspg/
aspirated tissue macrophages. They are small, round bodies 2–4 μm in diameter with indistinct
cytoplasm, a nucleus, and a small, rod-shaped kinetoplast.
 Culture:Growth of Promastigotes in days to weeks.Promastigotes are characterized by a flagellum
and a kinetoplast anterior to the nucleus. They are the infective stage to humans.

47. Typical eschar in given image is of:

A. louse-borne typhus
B. Scrub typhus
C. murine) typhus
D. None of these

Answer-B-scrub typhus
 It is a form of typhus caused by Orientia tsutsugamushi first isolated and identified in 1930 in Japan
 Scrub typhus is transmitted by some species of trombiculid mites ("chiggers", particularly
Leptotrombidiumdeliense), which are found in areas of heavy scrub vegetation.
 The bite of this mite leaves a characteristic black eschar that is useful for making the diagnosis.
 Scrub typhus is endemic to a part of the world known as the "tsutsugamushi triangle" (after the
name "Orientia tsutsugamushi" (formerly "Rickettsia tsutsugamushi"), the obligate intracellular
gram-negative bacterium causing same), which extends from northern Japan and far-eastern Russia
in the north, to the territories around the Solomon Sea into northern Australia in the south, and to
Pakistan and Afghanistan in the west.

48. Mw vaccine is made from which bacteria?

A. M. Welchii
For All Previous Year AIIMS PG Questions Visit

www.medicoapps.org/aiimspg/
B. M. Bovis
C. M. Indicus pranii
D. none of these

Answer-C- M. Indicus pranii


 Killed Mycobacterium indicus pranii (previously known as Mycobacterium w, popularly known as
Mw) vaccine has earlier been investigated in genital warts with encouraging results
 Mycobacterium w vaccine, a useful adjuvant to multidrug therapy in multibacillary leprosy.

49. All are seen in sickle cell anemiaexcept ?


A. Target cells
B. Malaria
C. Reticulocytosis
D. Jaundice

ed co
Answer-B- malaria

Apps
The possession of the sickle cell allele protects these individuals against malaria because the
parasite dies when potassium leaks out of RBCs as they become sickle shaped.
 Types,ofanemia that have protective effect against P. falciparum malaria : - G6PD deficiency, Sickle
cell anemia, Thalassemia, HbC, Pyruvate kinase deficiency
A Little Help to Get Started
50. Cerebral malaria is caused by?

A. Plasmodium falciparum
B. Plasmodium ovale
C. Plasmodium vivax
D. Plasmodium malariae

Answer- A-Plasmodium falciparum


 Plasmodium falciparum REF: Harrison's 17th edition, chapter 203
 Cerebral Malaria: Coma is a characteristic and ominous feature of falciparum malaria and, despite
treatment, is associated with death rates of 20% among adults and 15% among children. Any
obtundation, delirium, or abnormal behavior should be taken very seriously. The onset may be
gradual or sudden following a convulsion

For All Previous Year AIIMS PG Questions Visit

www.medicoapps.org/aiimspg/
Forensic Medicine

51. identify the given image of post mortem staining

A. Pallor mortis
B. Algor mortis
C. Postmortem hypostasis
D. Postmortem marbling

Answer- C- hypostasia-(suggligation)
 the accumulation of fluid or blood in the lower parts of the body or organs under the influence of
gravity, as occurs in cases of poor circulation or after death.
 hypostasis or suggillation, is the fourth stage of death and one of the signs of death. It is a settling
of the blood in the lower, or dependent, portion of the body postmortem, causing a purplish red
discoloration of the skin.
 When the heart stops functioning and is no longer agitating the blood, heavy red blood cells sink
through the serum by action of gravity. The blood travels faster in warmer conditions and slower in
colder conditions.

52. In the sequence in following clauses, which of the following is incorrect:

A. Second→ Permanent privaVon of the sight of either eye


B. First→ EmasculaVon
C. Eight→ Fracture or dislocaVon of a bone or tooth
D. sixth→ Permanent disfiguraVon of the head or face

For All Previous Year AIIMS PG Questions Visit

www.medicoapps.org/aiimspg/
Answer- C-Fracture or dislocation of a bone or tooth is the seventh clause of grievous hurt

Sec-320→ following kinds of hurt only are designated as "grievous":-


1. Firstly.-Emasculation
2. Secondly.-Permanent privation of the sight of either eye.
3. Thirdly.-Permanent privation of the hearing of either ear.
4. Fourthly.-Privation of any member or joint.
5. Fifthly.-Destruction or permanent impairing of the powers of any member or joint.
6. Sixthly.-Permanent disfiguration of the head or face.
7. Seventhly.-Fracture or dislocation of a bone or tooth.
8. Eighthly.-Any hurt which endangers life or which causes the sufferer to be during the space of
twenty days in severe bodily pain, or unable to follow his ordinary pursuits

53. Paradoxical undressing seen in


A. Immersion syndrome
B. Heat cramps

ed co
C. Hypothermia
D. Heat exhaustion
Apps
Ans- c- hypothermia

A Paradoxical
L i t t le Help to Get Started
undressing is a term for a phenomenon frequently seen in cases of lethal hypothermia.
Shortly before death, the person will remove all their clothes, as if they were burning up, when in
fact they are freezing.

54. What is the most common cause of parasuicide?


A. Drug ingestion
B. Hanging
C. Cutting wrist
D. Firearms

Ans. a. Drug ingestion

For All Previous Year AIIMS PG Questions Visit

www.medicoapps.org/aiimspg/
 Parasuicide is a suicide attempt or gesture and self-harm where there is no result in death. It is a
non7fatal act in which a person deliberately causes injury to him or herself or ingests any prescribed
or generally recognized therapeutic dose in excess.
 It is considered to be a serious public health issue. Parasuicide is the strongest known indicator for
a future successful suicide attempt.
 Examples of suicidal gestures include cutting, where the cut is not deep enough to cause significant
blood loss, or taking a non-lethal overdose of medication.'-
http://en.wikipedia.org/wiki/Parasuicide
 `Maximum cases of near-suicides are due to drug overdose. In India, wrist cutting is equally
common, but drug overdose is by far the most common cause of parasuicide in the world.'-
Methods used for parasuicide: results of the WHO/EURO Multicentre Study on Parasuicide.

55. Max damage is done by which Poison?


A. Irritant poison
B. Corrosive poison
C. Alcohol

ed co
D. Opioid

Ans. is'b'i.e., Corrosive poison Apps


 Corrosives fix, destroy and erode the surface with which they come in contact. Thus, they
A L i t t l e H e l p t o G e t S t a r t e d
cause extensive tissue damage.
Irritants produce less severe damage than corrosives and produces 'symPtoms simulating
gastroenteritis'

56. In
post mortem body traumatic injury of face with one eye missing and some injury on mouth
and nose ( right eye ball was missing , blood in socket area ) Cause of injury

A. Blunt rupture to eyeball


B. Evisceration by sharp weapon
C. Post mortem artefact
D. None of the Above

Answer B. Evisceration by sharp weapon

For All Previous Year AIIMS PG Questions Visit

www.medicoapps.org/aiimspg/
 In blunt rupture of eye the complete eye will not be missing. Also since the blood is present in the
socket so post mortem artifact is ruled out.
 Evisceration by Sharp object the eye ball will be completely missing. Some injury to mouth or nose
can be due to trauma in those areas.

Pharmacology

57.Preferred drug for the treatment of uncomplicated grade 2 hypertension in a 48 year old
man is

A. Chlorthalidone
B. Triamterene
C. Spironolactone
D. Furosemide


ed co
Ans. A. Chlorthalidone

therapy in the treatment of hypertension in the elderly population.


Apps
Thiazide diuretics are inexpensive, are generally well tolerated, and are recommended as a first-line

Thiazide diuretics are as effective as any drug for first-line treatment of hypertension in the elderly
A Little Help to Get Started
population is the Antihypertensive and Lipid-Lowering Treatment to Prevent Heart Attack Trial
(ALLHAT) study.
 Patients who received the diuretic had a lower incidence of CV events (secondary outcomes)
compared with the other groups. The diuretic treatment group had lower HF rates compared with
the CCB group

58. Propranolol is drug of choice for

A. Ulcerated infantile hemangioma


B. Lymphangioma
C. Pyogenic granuloma
D. Capillary malformation

Ans. A. Ulcerated infantile hemangioma


For All Previous Year AIIMS PG Questions Visit

www.medicoapps.org/aiimspg/
Ref: https://www.ncbi.nlm.nih.gov/pmc/articles/PMC3800297/
 Infantile hemangiomas (IHs) are the most common benign pediatric soft-tissue tumors.
 Ulceration—the most frequent complication of IH—tends to heal poorly and is associated with
pain, bleeding, infection, and scarring.
 Mainstay treatment modalities include propranolol (β-blocker) and corticosteroids, whose
effectiveness is countered by a need for long-term medication and risk of systemic adverse effects
and ulcer recurrence.

59. All are used for postpartum hemorrhage except


A. Misoprostol
B. Dinoprostone
C. Prostaglandin F2 alpha
D. Oxytocin

Ans. B. Dinoprostone

Dinoprostone:

A Naturally
ItLalso
ed co
Postpartum hemorrhage, the loss of more than 500 mL of blood after delivery, occurs in up to 18

i stimulates
t occurring
t l eosteoblasts
He l p factors
to G e bone
Apps
percent of births and is the most common maternal morbidity in developed countries.

t resorption
S t abyrosteoclasts.
prostaglandin E2 (PGE2). It has important effects in labour.
to release which stimulates ted
 As a prescription drug it is used as a vaginal suppository, to prepare the cervix for labour and to
induce labour.

Indication:
 For the termination of pregnancy during the second trimester (from the 12th through the 20th
gestational week as calculated from the first day of the last normal menstrual period), as well as for
evacuation of the uterine contents in the management of missed abortion or intrauterine fetal
death up to 28 weeks of gestational age as calculated from the first day of the last normal
menstrual period.
 In the management of nonmetastatic gestational trophoblastic disease (benign hydatidiform mole).
 Other indications include improving the cervical inducibility (cervical "ripening") in pregnant women
at or near term with a medical or obstetrical need for labor induction, and the management of
postpartum hemorrhage.

For All Previous Year AIIMS PG Questions Visit

www.medicoapps.org/aiimspg/
60. Most commonly implicated drug for acute liver failure is
A. Paracetamol
B. Valproate
C. Warfarin
D. Tetracyclines
Ans. A. Paracetamol

 Acute liver failure after administration of paracetamol at the maximum recommended daily dose
in adults.
 Paracetamol is the most commonly used analgesic and antipyretic in the world; it can be bought
without prescription in most countries despite being the commonest cause of acute liver failure in
western Europe.
 Prescribing information suggests that it is safe to use in adults in divided doses that total 4 g daily.
 Malnutrition, starvation, chronic alcohol misuse, and concomitant use of drugs that induce
cytochrome P450 enzymes increase the risk of hepatotoxicity induced by paracetamol.

61. There was

ed co
MRSA colonisation of anterior nares. What is the best treatment?

A. Topical bacitracin

AB. Oral
L iVancomycin
ttle Help to Get Started
Apps
an outbreak of MRSA in the hospital and it was found that a nurse of NICU had

C. Inhaled colistin
D. IV cefazolin

Ans. A. Topical bacitracin

 MRSA (methicillin-resistant Staphylococcus aureus) infection is a life-threatening bacterial infection


caused by Staphylococcus aureus that is resistant to the antibiotic methicillin. Severe cases usually
require hospitalization for treatment.
 MRSA infections are usually treated with systemic antibiotics, either orally or intravenously.
 Topical bacitracin in combination with neomycin and polymyxin B might be added to the therapy.

62. Fixed drug eruptions are frequently seen with?

For All Previous Year AIIMS PG Questions Visit

www.medicoapps.org/aiimspg/
A. Penicillin
B. Sulfonamide
C. Cetirizine
D. Roxithromycin

Ans. B.Sulfonamide

Drugs causing fixed drug eruption


 Paracetamol (Phenacetin)
 Sulfonamides
 NSAIDs
 Aspirin
 Barbiturates
 Dapsone
 Tetracyclines
 Phenylbutazone

All are topical hemostatic agents except?

ed co
63.

A. Bone wax & patty


B. HemCon bandage
Apps
C. Quikclot
AD. Fixclot
Little Help to Get Started
Ans. D. Fixclot

 Topical hemostatic agents are used when surgical hemostasis is inadequate or impractical. The
majority of routine, elective operations are performed in patients with normal hemostasis and with
minimal blood loss.
 The two main categories of topical hemostatic agents are physical agents, which promote
hemostasis using a passive substrate, and biologically active agents, which enhance coagulation at
the bleeding site

64. In iron poisoning drug Desferoxamine is given.what is its mechanism of action

A. By binding with trivalent Fe3+

For All Previous Year AIIMS PG Questions Visit

www.medicoapps.org/aiimspg/
B. -by inhibiting Hepcidin
C. -by inhibiting DMT1
D. -by inhibiting ferroportin

Ans. A. By binding with trivalent Fe3+


 Deferoxamine works in treating iron toxicity by binding trivalent (ferric) iron (for which it has a
strong affinity), forming ferrioxamine, a stable complex which is eliminated via the kidneys.
 100 mg of deferoxamine is capable of binding approximately 8.5 mg of trivalent (ferric) iron.
 Deferoxamine works in treating aluminum toxicity by binding to tissue-bound aluminum to form
aluminoxamine, a stable, water-soluble complex.
o The formation of aluminoxamine increases blood concentrations of aluminum, resulting in
an increased concentration gradient between the blood and dialysate, boosting the removal
of aluminum during dialysis.
 100 mg of deferoxamine is capable of binding approximately 4.1 mg of aluminum.

ed co
65. Abank employee felt depressed with no interest in activities came to AIIMS OPD. He was

Apps
started on Escitalopram. Which of these adverse effects cannot be explained with escitalopram?

A. Vivid dreaming
B. Anorgasmia
AC. Sialorrhea
Little Help to Get Started
D. Nausea

Ans. C. Sialorrhea

 Escitalopram belongs to a class of drugs known as selective serotonin reuptake inhibitors (SSRI).
 Improves your energy levels and feelings of well-being and decrease nervousness.
 Used to treat depression and anxiety.
 MOA:
o Works by helping to restore the serotonin balance in the brain.
 S/E:
o Headache, Nausea, Ejaculation disorder, Somnolence, Insomnia, Dry mouth, Constipation,
Fatigue, Libido decreased, Inability to achieve orgasm, Gas (flatulence), Toothache, Weight
gain, Menstrual disorder, Neck/shoulder pain, Runny nose & Flu-like syndrome.

For All Previous Year AIIMS PG Questions Visit

www.medicoapps.org/aiimspg/
66. Chlorpromazine act on which of the following receptors
1. D2 and 5HT2 receptors
2. GABA and Beta-adrenergic receptors
3. Muscarinic M1 and alpha-adrenergic receptors
4. H1 receptors

A. 1, 3 and 4 are correct


B. Only 2 is correct
C. 1 and 2 are correct
D. All are correct
Ans. A. 1, 3 and 4 are correct
Chlorpromazine:
 Chlorpromazine is a psychotropic agent indicated for the treatment of schizophrenia. It also exerts
sedative and antiemetic activity.
 Chlorpromazine has actions at all levels of the central nervous system-primarily at subcortical
levels-as well as on multiple organ systems.
 Chlorpromazine has strong antiadrenergic and weaker peripheral anticholinergic activity; ganglionic
blocking action is relatively slight.

ed co
 It also possesses slight antihistaminic and antiserotonin activity.
 Traditional antipsychotic agent with anti-emetic activity.
Apps
MOA:

A Exerts
L i itst antipsychotic
t l e effect H eby blocking
l p postsynaptic
t o dopamine
G e treceptors S tin cortical
areas of the brain, thereby preventing the excess of dopamine in the brain.
a r and te d
limbic

 This leads to a reduction in psychotic symptoms, such as hallucinations and delusions.


 Chlorpromazine appears to exert its anti-emetic activity by blocking dopamine receptors in the chemical
trigger zone (CTZ) in the brain, thereby relieving nausea and vomiting.

67. Whichof the following is/are adverse effect of SGLT2 inhibitors?


1. Ketoacidosis
2. Urosepsis
3. Fournier's gangrene
4. Angioedema

A. 1,2 and 4 are correct


B. 1, 2 and 3 are correct
C. Only 4 is correct

For All Previous Year AIIMS PG Questions Visit

www.medicoapps.org/aiimspg/
D. All are correct
Ans. B. 1, 2 and 3 are correct
Sodium glucose co-transporter-2:

 Sodium glucose co-transporter-2 [SGLT-2] present in proximal tubules.


 Glucose is freely filtered across glomerulus & is 100% reabsorbed in proximal tubules, via SGLT-2.

Sodium glucose co-transporter-2 inhibitors (SGLT-2 inhibitors):

 SGLT-2 inhibitors acts by inhibiting this transporter → Causing glucosuria in diabeVcs → causes
weight loss.
 Effective orally.
 Efficacy reduced in renal failure.

Side effects:
 Ketoacidosis, Urosepsi, Fournier's gangrene, Increased incidence of UTI & genital infections.

68.
ed co
positive patients.
A Little Help to Get Started
Reason: Frequent recurrence of genital herpes is seen in HIV patients
Apps
Assertion: Large doses of acyclovir are recommended for treating genital herpes in HIV

A. Both Assertion and Reason are true and Reason is correct explanation of Assertion
B. Assertion is true & reason is not the right explanation for assertion
C. Assertion is false & reason is also false
D. Both assertion & reason are false
Ans. A. Both Assertion and Reason are true and Reason is correct explanation of Assertion
ACYCLOVIR

 Antiviral drug.
 Requires virus-specific enzyme for conversion to active metabolite inhibiting DNA synthesis & viral
replication.
 Varicella-zoster viruses are not as sensitive as herpes simplex viruses.

INDICATIONS:

 Herpes simplex:
o For genital primary infections.
o Oral acyclovir - Faster healing rate than acyclovir ointment.
For All Previous Year AIIMS PG Questions Visit

www.medicoapps.org/aiimspg/
 For herpes simplex keratitis:
 Local/oral therapy successful.
 Herpes simplex encephalitis& herpes simplex neonatorum:
o Must be treated intravenously.
 Frequently relapsing herpes simplex (labial/genital):

o Long-term, prophylactic acyclovir administration.

69. Mention the true/false statements about digoxin toxicity?

A. Earliest manifestation of digoxin toxicity are gastrointestinal symptoms


B. Non specific vision changes may be noted in digoxin toxicity
C. Early toxicity may not correlate with serum levels
D. Neurological symptoms may occur without corresponding cardiovascular changes

Ans. All are true about digoxin toxicity.

DIGOXIN TOXICITY:

Features:


ed co
Generally unwell & lethargy.
A Little Help to Get Started


Nausea & vomiting.
Confusion.
Apps

 Yellow-green vision.
 Arrhythmias (e.g. AV block, bradycardia)
 Dizziness.

Precipitating factors:

 Renal disease
 Hypokalaemia
 Hypomagnesemia
 Hypoalbuminemia
 Hypothermia
 Hypothyroidism
 Hypercalcemia.
 Hypernatremia
 Acidosis.
 Myocardial ischaemia.
 Partial AV block.

For All Previous Year AIIMS PG Questions Visit

www.medicoapps.org/aiimspg/
Drugs:

 Amiodarone.
 Quinidine.
 Verapamil.
 Spironolactone.
 Furosemide.
 Hydrochlorothiazide -Compete with DCT secretion, hence reducing excretion.

Management

 Digibind.
 Correct ventricular arrhythmia by lignocaine.
o Bradyarrhythmias by propanolol.
o Atrial tachyarrhythmias by atropine.
Phenytoin.
Monitor K+

70. Mention the true/false statements about drug resistant malaria?

A D. Artemether
L i t t with
ed co
A. Not present in India
B. Quinine with clindamycin or doxycycline is still effective treatment
C. Chloroquine with sulfadoxine-pyrimethamine is effective

l elumefantrine
H eis luseful
p to Get Started
Apps
E. Monotherapy with artemisinin derivatives is not useful due to high relapse rate

Ans: B, D & E are True


A & C are false.
 Multidrug resistance of P. Falciparum is seen when the parasite is resistant to more than two
operational antimalarial compounds of different chemical classes and modes of action.
 Generally, the two classes first affected are the 4-aminoquinolines and the antifolates
(diaminopyrimidine, sulfonamides).
 Drug resistance results in a delay in or failure to clear asexual parasites from the blood, which
allows production of the gametocytes that are responsible for transmission of the resistant
genotype.

Drug resistance in India:


 In India, chloroquine-resistant P. falciparum malaria has been observed with increasing frequency
across the country in recent years.

For All Previous Year AIIMS PG Questions Visit

www.medicoapps.org/aiimspg/
 Considering this, the Government of India has recommended the combination of artesunate and
Sulfa-Pyrimethamine as the treatment of choice for P. falciparum (and mixed) infections all across
the country.
 In north east India, where resistance to SP has been documented, the National Drug Policy
recommends the use of Artemether and Lumefantrine for the treatment of P. falciparum malaria. P.
vivax remains sensitive to chloroquine all across the country.

71. Aspirin and phenobarbitone are acidic drugs whereas diazepam is a basic drug.
Mention the true/false statements about these drugs?

A. Aspirin is present in mainly non-ionised form in stomach, hence can be easily absorbed
B. Diazepam is mostly absorbed from intestine
C. Phenobarbitone can be absorbed from the stomach but most of the absorption occurs in small intestine due
to its large surface area
D. No drug is absorbed in the large intestine due to its very low surface area

ed co
E. Diseases decreasing the transit time of drugs like diarrhea will increase the drug absorption in the small

Ans:
intestine
Apps
A, B & C are True
D & E are false
A Little Help to Get Started
 Acidic drugs with pKa 3.5:
o Largely unionized at acid gastric pH & absorbed from stomach.
o Eg: Aspirin.
 Basic drugs with pKa 10:
o Largely ionized & are absorbed only in intestines.
o Eg: Atropine
 Unionized form is lipid soluble & ionized form is water soluble.
o Medium is same, hence drugs cross membrane.

Acidic drugs are lipid soluble (un-ionized form) in acidic medium.

 Eg: In acidic gastric pH these acidic drugs are more likely to be absorbed from stomach.
o Due to presence of unionized (lipid soluble) form.
 Hence, aspirin is more likely to be absorbed in stomach than morphine or atropine (basic drugs).
o This is reason for aspirin-induced gastric mucosal irritation.

Ion trapping:

 Unionized drugs reverts back to ionized form within cell (pH 7.0) before passing to ECF.
o Referred as “ion trapping”.

For All Previous Year AIIMS PG Questions Visit

www.medicoapps.org/aiimspg/
o Ie., Weak electrolyte crossing a membrane to encounter a pH from which it is not able to
escape easily.
 Basic drugs attain higher concentration intracellularly (pH 7.0 vs 7.4 of plasma).

72. Match the following drugs with their ocular adverse effects

Drug Adverse Effect


1. Amiodarone A. Blepharoconjunctivitis
2. Hydroxychloroquine B. Angle closure glaucoma
3. Systemic steroids C. Retinopathy
4. Digoxin D. Optic neuritis
E. Yellow vision
F. Cataract
G. Corneal microdeposits

A. 1-B, 2-D, 3-A, 4-C


B. 1-C, 2-F, 3-A, 4-E
C. 1-G, 2-C, 3-F, 4-E

ed co
D. 1-F, 2-G, 3-B, 4-C

Ans. C. 1-G, 2-C, 3-F, 4-E


Apps
 Amiodarone - Corneal microdeposits.
A Hydroxychloroquine
L i t t l e - Retinopathy
Help to Get Started
 Systemic steroids - Cataract
 Digoxin - Yellow vision.

73. Match the following drugs with organism they are used for
Drug Organism
1. Praziquantel A. Filaria
2. Diethylcarbamazine B. Giardia
3. Nitazoxanide C. Strongyloides
4. Mebendazole D. Tapeworms
E. Leishmania
F. Ascaris lumbricoides
A. 1-B, 2-D, 3-A, 4-C
B. 1-D, 2-A, 3-B, 4-F
C. 1-G, 2-C, 3-F, 4-E

For All Previous Year AIIMS PG Questions Visit

www.medicoapps.org/aiimspg/
D. 1-F, 2-G, 3-B, 4-C
Ans. B.1-D, 2-A, 3-B, 4-F
 Praziquantel - Tapeworms
 Diethylcarbamazine - Filaria
 Nitazoxanide - Giardia
 Mebendazole - Ascaris lumbricoides

74. Match the following anaesthetic agents with its appropriate property?

Drug Adverse Effect


1. Propofol A. Rigid chest syndrome
2. Fentanyl B. Pulmonary vasoconstriction
3. Midazolam C. Avoided in patients with egg Allergy
4. Nitrous oxide D. Hypotension
E. Dissociative anaesthesia
F. Adrenal insufficiency

ed co
A. 1-C, 2-A, 3-D, 4-B
B. 1-D, 2-A, 3-B, 4-F
C. 1-G, 2-C, 3-F, 4-E
Apps
D. 1-F, 2-G, 3-B, 4-C
A Little Help to Get Started
Ans. A. 1-C, 2-A, 3-D, 4-B

 Propofol - Avoided in patients with egg Allergy


 Fentanyl - Rigid chest syndrome
 Midazolam - Hypotension
 Nitrous oxide - Pulmonary vasoconstriction

75. Match the following drug poisonings with their antidotes


Drug overdose Antidote
1. HCN A. Trimethadione
2. Paracetamol B. Nalorphine
3. Morphine C. Bupropion
4. Nicotine D. Diazepam
E. N-acetylcysteine
F. Amyl nitrite
A. 1-B, 2-D, 3-A, 4-C
For All Previous Year AIIMS PG Questions Visit

www.medicoapps.org/aiimspg/
B. 1-D, 2-A, 3-B, 4-F
C. 1-G, 2-C, 3-F, 4-E
D. 1-F, 2-E, 3-B, 4-A

Ans .D. 1-F, 2-E, 3-B, 4-A


 HCN - Amyl nitrite
 Paracetamol - N-acetylcysteine
 Morphine - Nalorphine
 Nicotine - Trimethadione

Pathology

ed co
76. Best test that shows

A. bleeding time
B. aPTT
AC. prothrombin
L i t t time
le Help to Get Started
Apps
the integrity of intrinsic pathway of clotting mechanism

D. clotting time
Answer- C prothrombin time

The PT measures the time necessary to generate fibrin after activation of factor VII. It measures the
integrity of the "extrinsic" and "common" pathways (factors VII, V, X, prothrombin, and fibrinogen).

77. Patch test image is given and asked when will be readings taken

For All Previous Year AIIMS PG Questions Visit

www.medicoapps.org/aiimspg/
A. 24 hrs and 48hrs
B. 48 hrs and 96hrs
C. 48hrs and 72 hrs
D. 48hr & 96hrs

Answer- A24 hrs and 48hrs

There are 3 appointments, marks must be visible at the third appointment, usually 24–
24–48 hours later (72–
96 hours after application).

78. Gene which inhibits cell cycle is

A. p53
B. RB
C. p16
D. Notch receptor

Answer-

Cell cycle inhibitors

1. CIP/KIP family → p21, p27, p57

2. INK4a/ARF family → p161 NK4a, pl4ARF

79.Vascular event of inflammation in order-


order
immediate transient, delayed prolonged, immediate prolonged ,delayed prolonged
For All Previous Year AIIMS PG Questions Visit

www.medicoapps.org/aiimspg/
A. mast cell activation lead to release of histamine bradykinin
B. leukocytes mediated injury
C. sun exposure /heat stroke lead to mild endothelial injury
D. direct endothelial injury by bacterial toxin

Answer- A, C, D, B

80. Match the following

A. Dohle bodies- dilated rough ER


B. Auer rodes - DNA remnants
C. Flame cells - azurophilic
D. Howel jolly body- HbF
Answer-
A. Dohle bodies- dilated rough ER
B. Auer rodes- aurophilic
C. Flame cells- HbF
D. Howel jolly body- DNA remnants

81. Which process shows phases of acute inflammation-


A. Apoptosis
B. Pyroptosis

For All Previous Year AIIMS PG Questions Visit

www.medicoapps.org/aiimspg/
C. Necroptosis
D. Necrosis

Answer- D Necrosis

Direct injury to the endothelium causes cell necrosis and appearance of physical gaps at the sites of
detached endothelial cells.
In necrosis phospholipase activation occurs that leads to cell damage and leakage of enzymes outside the
cell is followed by inflammatory cells resulting in inflammation.

82. Histologic image of keratinising stratified squamous epithelium in stratum basale layer
which of the following cell is not present-

A. Langerhan cells
B. Melanocyte
C. Merkel's cells
D. Keratocyte

Answer- D. Keratocyte

Types of cells found within the stratum basale are melanocytes (pigment-producing cells), Langerhans cells
(immune cells), and Merkel cells (touch receptors).

83. All feature of reversible cell injury EXCEPT

A. ER SWELLING
B. DENS DEPOSITION OF MITOCHONDRIA
C. BLEB
D. Detachment of ribosome

Answer- B. DENS DEPOSITION OF MITOCHONDRIA


For All Previous Year AIIMS PG Questions Visit

www.medicoapps.org/aiimspg/
The ultrastructural charges (seen on electron microscopy) are :-

1. Plasma membrane alterations → Blebbing blunVng loss of microvilli.


2. Mitochondrial changes → Swelling, small amorphous densities.
3. Dilatation of ER and detachment of ribosome
4. Nuclear alterations

84. Which of the following Glomerulonephritis has Nephrotic syndrome except


A. FSGS
B. Post-infectious Glomerulonephritis
C. MPGN
D. Minimal Change Disease

Answer- B. Post-infectious Glomerulonephritis

 Focal segmental glomerulosclerosis is characterised by a sclerosis of segments of some glomerules.


It is likely to present as a nephrotic syndrome.
 Membranous glomerulonephritis may cause either nephrotic or a nephritic syndrome.
Post-infectious glomerulonephritis- present with malaise, a slight fever, nausea and a mild nephritic

ed co

syndrome of moderately increased blood pressure, gross haematuria, and smoky-brown urine.

with the nephritic syndrome, hypocomplementemia. Apps
Membranoproliferative GN (MPGN), also known as mesangiocapillary glomerulonephritis- present

A Little Help to Get Started


85. Chimerism phenomenon is associated with which of the following-

A. Paternity test
B. Maternity test
C. Person identification test
D. organ transplantation case

Answer- D. organ transplantation case

Chimerism can occur in animals is by organ transplantation, giving one individual tissues that developed
from a different genome. For example, transplantation of bone marrow often determines the recipient's
ensuing blood type.

86. Correct about regulatory T - cell?


For All Previous Year AIIMS PG Questions Visit

www.medicoapps.org/aiimspg/
A. Release of granzyme A or B by the TREG which induces apoptosis of target T cells
B. CD 8 associated with suppressive cells
C. CD 4 & CD 25 marker
D. All are true

Answer- C. CD 4 & CD 25 marker

Tregs express the biomarkers CD4, FOXP3, and CD25.

87. Example of Dystrophic calcification-

A. Hyperparathyroidism
B. Sarcoidosis
C. Hypervitaminosis D
D. Myostitis ossificans

Answer- D. Myostitis ossificans

ed co
Myositis Ossificans (MO) is an unusual pathological entity still largely unknown, characterized by

Apps
dystrophic calcification leading to heterotopic ossification of intramuscular connective tissue.

88. Arrange them in the order of their size of lesions according to depth-
AA. Bullous
L i tPemphigoid
tle Help to Get Started
B. Epidermolysis Bullosa
C. Pemphigous Vulgaris
D. IgA pemphigus

Answer- C, D, A, B

C. Pemphigus vulgaris
D. IgA pemphigus
A. Bullous pemphigoid
B. Epidermolysis bullosa

89. Amyloid protein in Hemodialysis associated with amyloidosis is-

A. AA

For All Previous Year AIIMS PG Questions Visit

www.medicoapps.org/aiimspg/
B. A ઺
C. ઺- 2 microglobulin
D. Transthyretin

Answer-C. ઺- 2 microglobulin

Hemodialysis-associated amyloidosis-
Associated disease is Chronic renal failure
Major fibril protein is A઺2m
Clinically related precurosr protein- Beta- 2 microglobulin

90. Under polarised light, the congo red stained amyloid shows-

A. Pink or red color

B. White color

ed co
C. Apple green birefringence
D. None

Answer- C. Apple green birefringence


Apps
Congo red staining shows an apple-green birefringence under polarized light, a diagnostic feature of
A Little Help to Get Started
amyloid.

91. In a patient with Hepatitis B infection. Which one of the markers will be increased?

A. HbsAg

B. HbcAg
C. Anti HbsAg IgG
D. Anti HbcAg IgG

Answer- D. Anti HbcAg IgG

 Best marker for diagnosing acute hepatitis B is IgM anti-HBc as it is found only in the acute phase of
hepatitis B (In chronic hepatitis IgG anti-HBc is found).

For All Previous Year AIIMS PG Questions Visit

www.medicoapps.org/aiimspg/
92. Medlar bodies are found in

A. Sporotrichosis
B. Chromoblastomycosis
C. Mycetoma
D. Histoplasmosis

Answer- B. Chromoblastomycosis

Medlar bodies, also known as sclerotic or muriform cell.


When present in skin or subcutaneous tissue, the cells are indicative of chromoblastomycosis.

ed co
A Little Help to Get Started
Apps
ENT

93. The following statements about thyroglossal cyst are true, EXCEPT:

A. The cyst is located within 2 cm of the midline

B. Incision and drainage is the treatment of choice


C. The swelling moves upwards on protrusion of tongue
D. It is the frequent cause of anterior midline neck masses in the first decade of life

Ans: B. Incision and drainage is the treatment of choice

Thyroglossal cyst is a fluid filled sac resulting from a persistent thyroglossal duct.
Excision of the thyroglossal cyst along with the track and the body of the hyoid bone is the treatment of
choice (Sistrunk’s operation).

For All Previous Year AIIMS PG Questions Visit

www.medicoapps.org/aiimspg/
Ref: Problem Based Approach in Pediatric Surgery By Rao, Page 108; Bailey and Love’s Short Practice of
Surgery, 24th Edition, Pages 777-78.

94. female patient came with right side hearing loss, better heard in a noisy
environment,Audiogram shown with about 30-40 dB gap between AC-BC of right & left ear.
Rinne's test negative, Weber's test centralised. Which of the following condition shown?

A. Ménière's disease
B. stapedial otosclerosis
C. Presbycusis
D. Vestibular schwannoma

Answer-B- stapedial otosclerosis

 In otosclerosis the normal dense endochondral layer of the bony otic capsule gets replaced by
irregularly laid spongy bone. Most common site is fissula ante fenestram (anterior to

ed co
 the oval window)
 Age group affected is 20-45 years (maximum between 20-30 years). Male:female ratio is 1:2.


Apps
meniere's diseasei.e Endolymphatic hydrops leads to SNHL and not conductive hearing loss.
Tuning ForkTests in Otosclerosis: Rinnes test-negative ,Weber's test-lateralized to ear with greater
conductive loss.

A Little Help to Get Started

Ophthalmology
95. First order neuron of visual pathway

A. Photoreceptor
B. bipolar neuron
C. lateral geniculate body
D. all of the above

Answer-:A: photoreceptor

 Rods and cones are the first-order receptor cells that respond directly to light stimulation.
For All Previous Year AIIMS PG Questions Visit

www.medicoapps.org/aiimspg/
 Bipolar neurons are the second-order neurons that relay stimuli from the rods and cones to the
ganglion cells.
 Ganglion cells third-order neurons that form the optic nerve (CN II).

96. IOL placed in the young male., After 10 year what should be done..?

A. IOL should be removed in case of Posterior Capsule ossification [PCO]


B. Never be removed
C. Remove when presbyopia sets in
D. Should be changed after 10 year

Answer A. IOL should be removed in case of Posterior Capsule ossification [PCO]

1. The indications for removing an intraocular lens (IOL) are:


 Chronic uveitis
 Endothelial corneal dystrophy
 Uncontrollalbe glaucoma
 Metal loop cutting pupillary sphincter
 Gross decentration of IOL (fibrous bands)
 Extraocular dislocation of IOL G. Recurrent severe hyphema
 Development of rubeosis iridis
 Removal of iris (1) Iris tumor (2) Epithelial downgrowth
 Endophthalmitis
 Unilateral IOL in pending bilateral aphakia

2. The following may be indications for removing and/or replacing and/or replacing an IOL:
 Wrong dioptric power

For All Previous Year AIIMS PG Questions Visit

www.medicoapps.org/aiimspg/
 Foreign body attached to IOL
 IOL covered with pigment
 Repair retinal detachment after extracapsular cataract extraction
 Choyce lens too short
 Dannheim IOL with absorbed supramid loop tips
 Dislocated Ridley IOL
 Sclero-conjunctival erosion of Strampelli's "external-fixation" IOL loop

Reference - https://www.ncbi.nlm.nih.gov/pubmed/904866

97. The given image is related to following?

A. Marcus Gunn pupil


B. Optic neuritis
C. Robertson-Argyll pupil
D. Pin point pupil

Answer-A-Marcus gun pupil

 RELATIVE AFFERENT PATHWAY DEFECT (RAPD) OR MARCUS GUNN PUPIL It is the paradoxical
response of a pupil to light in the presence of a relative afferent pathway defect (RAPD). Caused by
an incomplete optic nerve lesion or a severe retinal disease. It is best tested by ‘swinging flashlight
test’.
 Normally, both pupils constrict equally and the pupil to which light is transferred remains tightly
constricted.
 In the presence of RAPD in one eye, the affected pupil will dilate (paradoxical response) when the
flashlight is moved from the normal eye to the abnormal eye.
 This response is called Marcus Gunn Pupil.
 It is the earliest indication of optic nerve disease even in the presence of a normal visual acuity

For All Previous Year AIIMS PG Questions Visit

www.medicoapps.org/aiimspg/
98. The Following test is used for

A. Streopsis Test
B. ColorVIsion
C. Strabismus
D. D> All of the above

Answer A. Streopsis Test

 Stereopsis is the ability to perceive depth and 3- d structure obtained on the basis of visual
information deriving from two eyes by individual with normally developed binocular vision •
stereopsis is usually reserved for the impression of depth arising from binocular disparity
 stereopsis is measured at near as part of clinical assessment of patients disorders of ocular motility
and strabismus • stereopsis can be measured near and distance

99. on examination 18 month old child has inward deviation of both eyelids,which of the
following test should be done?

A. Forced duction test


B. Cover Uncover Test
C. Fundus Examination
D. All of the above

Answer- B- Cover Uncover Test

Since the diagnosis is strabismus the correct answer is B


For All Previous Year AIIMS PG Questions Visit

www.medicoapps.org/aiimspg/
The forced duction test is performed in order to determine whether the absence of movement of the eye
is due to a neurological disorder or a mechanical restriction. The anesthetized conjunctiva is grasped with
forceps and an attempt is made to move the eyeball in the direction where the movement is restricted.

Since the diagnosis is strabismus the correct answer is B

100. A patient with a history of diabetes for one year with no other complications should
have an ophthalmic examination?

A. As early as feasible
B. After 5 years
C. After 10 years
D. Only after visual symptoms level

ed co
Answer-6- A i.e. As early as feasible


screened (visual activity measurement and fundus examination by ophthalmoscopy)
Apps
All diabetic (IDDM & NIDDM both) aged over 12 years and/or entering puberty should be

 For retinopathy. and those with risk for visual loss referred to an ophthalmologist.
 Type I DM (IDDM) require ophthalmoscopic examination within 3 years of diagnosis and annual
A Little Help to Get Started
review. (If lt is diagnosed before the age of puberty).
 Type II DM (NIDDM) require ophthalmoscopic examination at the time of diagnosis (because it
is usually diagnosed after the age of 12 years) and annual review.

101. Image nail piercing into the cornea of the eye which of the following is not done at first:

For All Previous Year AIIMS PG Questions Visit

www.medicoapps.org/aiimspg/
A. primary survey
B. testing visual acuity
C. removal of foriegn body
D. antibiotics

Answer C. removal of foriegn body

Removal of The nail should not be done at first rather the patient should be immediately referred to an
ophthalmologist for further management.

Reference - https://www.ncbi.nlm.nih.gov/pmc/articles/PMC4790160/

PSM

102.Government initiative to improve the facilities in labour room in all govt hospitals is under

A. Ayushman Bharat Scheme


B. Laqshya
C. Newborn delivery room program
D. Janani suraksha yojana

Ans.B. Laqshya

‘LaQshya’ programme of the Ministry of Health and Family Welfare aims at improving quality of care in
labour room and maternity Operation Theatre (OT).

Objective:
For All Previous Year AIIMS PG Questions Visit

www.medicoapps.org/aiimspg/
 To reduce maternal and newborn mortality & morbidity due to APH, PPH, retained placenta,
preterm, preeclampsia & eclampsia, obstructed labour, puerperal sepsis, newborn asphyxia, and
sepsis, etc.
 To improve Quality of care during the delivery and immediate post-partum care, stabilization of
complications and ensure timely referrals, and enable an effective two-way follow-up system.
 To enhance satisfaction of beneficiaries visiting the health facilities and provide Respectful
Maternity Care (RMC) to all pregnant women attending the public health facility.

103. Which of the following are the responsibilities of ASHA worker ?

A. Making slides of Malaria


B. Accompanying the pregnant to the Labour room
C. To generate demand on key determinants of health
D. Dot Providers of Directly Observed Treatment Short-course

Ans. B. C. D.

ROLES OF ASHA WORKERS:



ed co Apps
ASHA create awareness and provide information to the community on determinants of health such
A Little Help to Get Started
as proper diet and nutrition, basic sanitation and hygienic practices, healthy living and working
conditions, information on existing health services and the need for timely utilization of health and
family welfare services at doorsteps.

 Conduct home visits of the pregnant women/mother/newborn under Home Based Post Natal Care
(HBPNC), and they are supposed to counsel pregnant women on birth preparedness, importance of
safe delivery, breastfeeding and complementary feeding, immunization, contraception and
prevention of common infections including Reproductive Tract Infection/Sexually Transmitted
Infection (RTIs/STIs) and care of the young child.

 Mobilize the community and facilitate them in accessing health and health related services
available at the village/sub-center and other public health facilities, such as Routine Immunization
(RI), Ante Natal Check-ups (ANCs), Post Natal Check-ups (PNCs), sanitation and other services being
provided by the Government. ASHA have to work with the Village Health Sanitation Nutrition
Committee/Village Level Committee (VHSNC/VLC) of the Gram Panchayat to facilitate a
comprehensive village health plan with ANM, AWWs and PRI members.

 Mobilize targeted community once in a month for the celebration of Village Health Nutrition Days
(VHND) at their Aanganwadi Centre.

For All Previous Year AIIMS PG Questions Visit

www.medicoapps.org/aiimspg/
 ASHA have to arrange escort/accompany pregnant women & children requiring treatment/
admission to the nearest pre- identified health facility i.e. Primary Health Centre/Community Health
Centre/ First Referral Unit (PHC/CHC /FRU).

 ASHA have to provide primary medical care for minor ailments such as diarrhoea, fevers, and first
aid for minor injuries.

 Dot Providers of Directly Observed Treatment Short-course (DOTS) under Revised National
Tuberculosis Control Programme (RNTCP).

 ASHA are also acting as a Depot Holder for essential provisions being made available to every
habitation like Oral Rehydration Therapy (ORS), Iron Folic Acid Tablet (IFA), chloroquine, Disposable
Delivery Kits (DDK), Contraceptives (Condoms, Oral Pills, Emergency Pills), etc. Provision of Drug Kit
and HBPNC Kit has been made for ASHA. Contents of the Drug/HBPNC Kits are based on the
recommendations of the expert/technical advisory group set up by the Govt of India.

 ASHA are expected to provide first information about the births and deaths in her village and any
unusual health problems/disease outbreaks in the community to the SC/PHC/CHC or directly to the
District Authorities or even to the State HQ at the NRHM Help Line.

A.
B.

D.
ed co
104.Biomedical waste management in

Chemical waste
Syringe Caps
Match the following:

AC. L i t t l e H e l p t o G e t S t a r t e d
Scalpel
Glasswares
Apps

E. Gloves
I. Category No.4
II. Category No.6
III. Category No.7
IV. Category No.10

Answer:
A. - IV.
B. -VII
C. -IV
D. -I
E. II

For All Previous Year AIIMS PG Questions Visit

www.medicoapps.org/aiimspg/
HOSPITAL WASTE CATEGORIES:

Categories of bio-medical waste in India

1 Human anatomical waste


2 Animal waste
3 Microbiology and biotechnology waste
4 Waste sharps (used and unused needles, syringes, scalpels etc)
5 Cytotoxic drugs and discarded medicines
6 Solid waste (contaminated with blood and fluids including cotton, dressings, soiled plasters,
beddings)
7 Solid waste (other than waste sharps such as rubber tubes, catheters and I.V sets)
8 Liquid waste
9 Incineration ash
10 Chemicals used in disinfection, insecticides

COLOR CODING & TYPES OF CONTAINER:

Colour coding Type of Container Waste Category Treatment options


Yellow Plastic Bags Human and animal wastes, Incineration/ Deep Burial

ed co
Microbial and Biological wastes
and soiled wastes
(Cat 1,2,3 and 6) Apps
Red Disinfected Microbiological and Biological Autoclave/ Microwave/
container/ Plastic wastes, Soiled wastes, Solid Chemical Treatment)
A Little Help to Get Started
bags wastes
(Cat 3,6,7)
Blue/ White/ Plastic bag, Waste sharps and solid waste Autoclave/ Microwave/
Transparent Puncture proof ( Cat 4 &7) Chemical Treatment
container Destruction and Shredding
Black Plastic bag Discarded medicines, Cytotoxic Disposal in secured land fills
drugs, Incineration ash and
chemical waste
(Cat 5,9 & 10)
Green Plastic Container General waste such as office Disposed in secured landfills
waste, food waste & garden
waste

105. Immunization that could help post disaster includes -

A. Measles

For All Previous Year AIIMS PG Questions Visit

www.medicoapps.org/aiimspg/
B. Cholera
C. Typhoid
D. All

Ans. A. Measles

Vaccines recommended in disasters


 Following vaccines are recommended
1. Children < 10 years :- DPT, inactivated polio (IPV), H.influenzae type b (Hib), hepatitis B,
pneumococcal conjugate vaccine (PCV), measles-mumps-rubella (MMR), varicella vaccine,
influenza, hepatitis A and rotavirus.
2. Children and adolescents (11-18 years):- Tetanus, diphtheria, pertussis, meningococcal conjugate
vaccine (MCV), Influenza.
3. Adults (>18 years):- Tetanus, diphtheria, pertussis, pneumococcal polysaccharide vaccine (PPSV23),
and influenza.
 Vaccination against typhoid and cholera is not recommended.

ed co
106.Ina village of 20,000 population 456 births occurred in april, 56 showed no signs of life at
Apps
birth, 56 died before 28 days of life, 34 died between 28 days- 1 year, 500 was total number
of deaths in that year. Infant mortality rate?

A. 197.4
AB. 320
Little Help to Get Started
C. 225
D. 125

Ans.C.225

IMR = Number of deaths of children less than 1 year of age in a year X 1000/number of live births in the
same year.
IMR=56+34 X1000/400= 225

107.AStudy conducted in a population.Diastolic Blood Pressure mean 110 mm Hg with SD of 11


mm Hg, Vitamin D mean 18 ng/ml with SD of 3 ng/ml. what is the relation between the two
variance
For All Previous Year AIIMS PG Questions Visit

www.medicoapps.org/aiimspg/
A. Variance of Vitamin D > DBP
B. Variance of DBP > 4 times Vitamin D
C. Data insufficient to comment upon
D. None

Ans.b). Variance of DBP > 4 times Vitamin D

 Coefficient of Variance (CV) is a tool to compare variability of two different characteristics (eg. BP,
serum creatinine, height weight etc.) in the same group of subjects or compare variability of the
same character in two or more different groups.
 Thus it is a measure used to compare relative variability.
 For example, coefficient of variance measures
1. whether weight varies more than height in a group of student
2. whether weight varies more in girls of boys.

Calculation:
 CV= standard deviation x 100/ mean
 For Blood pressure- 110x100/11=1000
 For Vitamin D- 3X100/18= 16.7

108.Broken

A A. Red
ed co
ampula is thrown into which coloured bin?

Little Help to Get Started


B. Yellow
Apps

C. Blue
D. Black

Ans. Blue

COLOR CODING & TYPES OF CONTAINER:

Colour coding Type of Container Waste Category Treatment options


Yellow Plastic Bags Human and animal wastes, Incineration/ Deep Burial
Microbial and Biological wastes
and soiled wastes
(Cat 1,2,3 and 6)
Red Disinfected container/ Microbiological and Biological Autoclave/ Microwave/
Plastic bags wastes, Soiled wastes, Solid Chemical Treatment)
wastes
(Cat 3,6,7)
For All Previous Year AIIMS PG Questions Visit

www.medicoapps.org/aiimspg/
Blue/ White/ Plastic bag, Puncture Waste sharps and solid waste Autoclave/ Microwave/
Transparent proof container ( Cat 4 &7) Chemical Treatment
Destruction and Shredding
Black Plastic bag Discarded medicines, Cytotoxic Disposal in secured land
drugs, Incineration ash and fills
chemical waste
(Cat 5,9 & 10)
Green Plastic Container General waste such as office Disposed in secured
waste, food waste & garden landfills
waste

109.Case control study was done regarding breast cancer & risk factors & odds ratio was
obtained. Which of the following are causative factors ?

Parameter Case Control OR


Age at menarche>12 1714 2329 1.04
Menopause 1771 2408 1.53
OCP use 935 1105 1.02

ed co
Smoking 93 214 0.97
Family history
BMI>27.5
513
165
502
493
Apps
1.10
0.53
Breastfeeding 942 2514 0.56
A L i t t l e 2778H e l p t o
Multiparity(2) 3366 G e t S 1.05
tarted
A. 6
B. 5
C. 3
D. 2

Ans.B) 5

 An odds ratio (OR) is a statistic that quantifies the strength of the association between two events,
A and B.
 Two events are independent if and only if the OR equals 1: the odds of one event are the same in
either the presence or absence of the other event.
 If the OR is greater than 1, then A and B are associated (correlated) in the sense that, compared to
the absence of B, the presence of B raises the odds of A, and symmetrically the presence of A raises
the odds of B.
 Conversely, if the OR is less than 1, then A and B are negatively correlated, and the presence of one
event reduces the odds of the other event.
For All Previous Year AIIMS PG Questions Visit

www.medicoapps.org/aiimspg/
 Most Importantly Odds ratio does not demonstrate causality .
 But we can say that they are positively correlated. Since 5 Factors have odds ratio more than 1 so
the answer will be B

110.A study is to be conducted with regards to the fat content in the expressed breast milk of
pre-term infants as compared to term infants. Which study design is best suited?

A. Case control
B. Prospective cohort
C. Longitudinal study
D. Ambispective

Ans. Since there is a comparison between fat content of EBM in preterm vs term infant the best study
design will be Case cotrol.

Prospective Cohort study will be appropriate if we want to see the change in the fat content of EBM in

ed co
preterm infants.

Apps
111.Key indicator for AFP surveillance -.

AA. AtLleast
i tonet case
l eof non-polio
H eAFPlperpyear per
to G e oftunderS5 years
1000 population tarted
B. At least one case of non-polio AFP per year per 100000 population of under 5 year
C. At least one case of non-polio AFP per year per 1000 population of under 15 years
D. At least one case of non-polio AFP per year per 100000 population of under 15 years

Ans.d)At least one case of non-polio AFP per year per 100000 population of under 15 years

The number of AFP cases reported each year is used as an indicator of a country's ability to detect polio,
even in countries where the disease no longer occurs.

Polio surveillance

 It is the most important part of whole polio eradication intiative. It has two components:-
 Acute flaccid paralysis (AFP) surveillance
 Acute flaccid paralysis is defined as acute onset (< 4 weeks) of flaccid paralysis (reduced tone)
without other obvious cause in children WHO recommends the immediate reporting and
investigation of every case of AFP in children less than 15 years.

For All Previous Year AIIMS PG Questions Visit

www.medicoapps.org/aiimspg/
112. A graph given. Calculate regression equation.

A. X = 2Y + X
B. X = Y + 2*X
C. X = Y/2
D. Y = X/2

Answer C. X = Y/2

The Values of X, Y are


5,10
10,20
15, 30
20,40 ..

So The Correct Regression equation is X = Y/2

113.In assessing the association between maternal nutritional status and the birth weight of the
newborns, two investigators A and B studied separately and found significant results with p
values 0.02 and 0.04 respectively. From this information, what can you infer about the
magnitudes of association found by the two investigators -

A. The magnitude of association found by Investigator A is more than found by B


B. The magnitude of association found by Investigator B is more than found by A
C. The estimates of association obtained by A and B will be equal, since both are significant
D. Nothing can be concluded as the information given is inadequate

Ans. D. Nothing can be concluded as the information given is inadequate

For All Previous Year AIIMS PG Questions Visit

www.medicoapps.org/aiimspg/
114.Looking at the scatter diagram below What is the relation between two graphs?

A. Positive correlation between birth weight and gestational age


B. Negative correlation between birth weight and gestational age
C. No correlation between birth weight and gestational age
D. he data is insufficient to comment

Ans. A. Positive correlation between birth weight and gestational age

Scatter plot of bone SOS vs gestational age, r=0.569, p<0.001 (y=2161+26.2x). (b) Scatter plot of bone SOS
vs birth weight, r=0.420, p<0.001 (y=2800+0.113x).
This shows a significant (p≤0.001) positive correlations between SOS and gestational age, birth weight,
length, head circumference and tibial length.

115. Matching is method to eliminate confounding bias. It is used in-

A. Case control study


B. Cohort study
C. Experimental study
D. Cross sectional study

Ans. A. Case control study

For All Previous Year AIIMS PG Questions Visit

www.medicoapps.org/aiimspg/
 Matching is most useful in case control studies (though it can also be used in cohort studies, but
benefit is not assured if exposure is not randomized).

Method Used to Control Confounding

Method Utility to control confounding

Randomization Most ideal method

Restriction Limiting study to people who have particular characteristics

Matching Mostly useful in case control studies

Stratification Useful for larger studies

When many confounding variables exist simultaneously


Statistical modelling

116.Test used to assess quantitative observations before and after an intervention -

A.

B.

C. ed co
Unpaired T-test
Paired T-test
Chi-square test

AD. Fisher-T-test
Little Help to Get Started
Apps

Ans. B.Paired T-test

Paired test : Is applied to paired data, when each individual gives a pair of observations such as : when
observations are made before and after the play of a factor e.g. pulse rate before and after a drug. Further,
it proceeds similar to the unpaired test.

117.Transmission assessment survey (TAS) is done in the following?

A. To determine when infections have been reduced below these target thresholds
B. For assessing primary immunization coverage
C. To provide reliable estimates of birth rate, death rate and infant mortality rate
D. All

Ans.A.To determine when infections have been reduced below these target thresholds

For All Previous Year AIIMS PG Questions Visit

www.medicoapps.org/aiimspg/
 WHO recommends the transmission assessment survey (TAS) to determine when infections have
been reduced below these target thresholds and MDA can stop.
 Once MDA has stopped, TAS is used as a surveillance tool to determine that infection levels are
sustained below target thresholds.
 Mass drug administration (MDA) is needed to reduce infection in the community to levels below a
threshold at which mosquitoes are unable to continue spreading the parasites from person to
person and new infections are prevented.

Medicine

118.Young patient with fever, arthralgia,headache and is on NSAIDs,and has macules over the
nose image is given

A. Chikungunya
B. Measles
C. Dengue
D. Fixed drug reactions

Answer- A. Chikungunya

Symptoms of the disease include:


 Fever
 A petechial or maculopapular rash of the trunk and occasionally the limbs
 Arthralgia or arthritis affecting multiple joints.
 The pain associated with CHIKV infection of the joints persists for weeks or months.
 Typical macules over the nose.
 Typically, the fever lasts for two days and then ends abruptly.
 Ocular inflammation may present as iridocyclitis and have retinal lesions as well.
 Pedal edema (swelling of legs) is observed in many patients.

For All Previous Year AIIMS PG Questions Visit

www.medicoapps.org/aiimspg/
119. Which score is used for wound infection

A. Glascow coma scale


B. Southampton score
C. Apgar score
D. SIRS score

Answer- B. Southampton score

Southampton score and ASEPSIS criteria are used for wound infection.

120.causes of thyroid storm

A. ineffective pre-operative hormonal control


B. manhandling of the thyroid gland during surgery

L i t t reason
ed co
C. parathyroid tail left after surgery
D. Poor Preoperative preparation of the patient

Answer- D. Poor Preoperative preparation of the patient


A Commonest l e for thyroid
H estorml pis poortpreooperative
Gpreparation
e t Sof thet patient.
arted
Apps
 Acute illness (stroke, infection, trauma, diabetic ketoacidosis)
 Surgery
 Inadequate control of hyperthyroidism
 Radioactive treatment of hyperthyroidism

121. If the cycle starts with ' a ' wave of jvp . Arrange the following in order-

A. R wave
B. T wave
C. 1st heart sound
D. Rapid ejection phase

Answer- C, A, B, D

For All Previous Year AIIMS PG Questions Visit

www.medicoapps.org/aiimspg/
The "a" wave is approximately synchronous with the first heart sound (S1) and just precedes the carotid
upstroke.
Shortly after the a-wave there is a second peak , the c-wave. The c-wave immediately follows the r wave
of the ECG waveform.
The v-wave corresponds to the end of the t wave in the ECG waveform.

122. Which of the following doesn't have any effect on pancreatic secretion?

A. Cck
B. Gastrin
C. Secretin
D. Gastric inhibitory polypeptide

Answer- D. Gastric inhibitory polypeptide

CCK- As chyme floods into the small intestine, cholecystokinin is released into blood and binds to

ed co
receptors on pancreatic acinar cells, ordering them to secrete large quantities of digestive enzymes.
Secretin-
Apps
The predominant effect of secretin on the pancreas is to stimulate duct cells to secrete water and
bicarbonate.
Gastrin- Stimulate acid secretion by the parietal cell, gastrin stimulates pancreatic acinar cells to secrete
digestive enzymes.
A Little Help to Get Started
123.A young man was on high protein diet. After 3 days he developed weakness. Blood
investigation revealed hypoglycemia. Hypoglycemia is due to inhibition of which of the following
enzymes

A. Glucose 6 phosphatase
B. Glycogen phosphorylase
C. Phosphoglucomutase
D. PEP carboxylase
Answer- D. PEP carboxylase

124. What is the most common tumor of mediastinum?

For All Previous Year AIIMS PG Questions Visit

www.medicoapps.org/aiimspg/
A. Neurogenic
B. Thymoma
C. Lymphoma
D. Metastatic tumor

Answer- A. Neurogenic

the most common mediastinal masses are neurogenic tumors (23%), thymomas (21%), lymphomas (13%)
and germ cell tumors (12%). Mediastinal masses are most frequently located in the anterosuperior
mediastinum (54%), with the posterior (26%) and middle mediastinum (20%) being less frequently involved

125.In which of the following tumor MRI is better than CT Scan..

A. Brochogenic Carcinoma
B. SCC of lung

ed co
C. Mesothelioma
D. Pancosttumor

Answer- D. Pancosttumor
Apps
MRI: An MRI is generally more accurate than a CT scan at uncovering the extent to which a tumor has
A Little Help to Get Started
invaded other structures

126.Which of the following has intracellular receptor

A. Glucagon
B. Insulin
C. Epinephrine
D. Thyroxine

Answer- D. Thyroxine

Classic hormones that use intracellular receptors include thyroid and steroid hormones.

For All Previous Year AIIMS PG Questions Visit

www.medicoapps.org/aiimspg/
127. 45 years old woman, Hb-8.9 gm/dl, TLC-3,000 Platelets-94,000, Spleen-3 cms. palpable,
which of the following is least likely?

A. Primary myelofibrosis
B. Myelodysplastic syndrome
C. Myeloproliferative disorders
D. Acute Myeloid Leukemia
Answer-

128.A patient presents to the ER after a RTA with multiple rib injuries. He is conscious, speaking
single words. RR = 40/min, BP= 90/40 mmHg. What is the next immediate step in management?

A. Intubate the patient


B. Urgent fluid infusion
C. Chest X ray
D. Needle insertion in 2nd ICS

Answer- D. Needle insertion in 2nd ICS

This is a case of tension pneumothorax. Although the new ATLS update is 5th intercostal space in mid
maxillary line, but in this question we will go with a time tested method of needle in 2nd intercostal space.

129. Name the 2nd and 3rd marked area in the diagram-

For All Previous Year AIIMS PG Questions Visit

www.medicoapps.org/aiimspg/
A. Pulmonic area & Tricuspid area
B. Erb’s point and Pulmonic area
C. Tricuspid area & Mitral area
D. Aortic area & Mitral area

Answer- C. Tricuspid area & Mitral area

130. Identify the disease in the image?

A. Cutaneous larva margins


B. Scabies
C. Psoriasis
D. None

For All Previous Year AIIMS PG Questions Visit

www.medicoapps.org/aiimspg/
Answer- A. Cutaneous larva margins

The most common species causing this disease is Ancylostomabraziliense.

131.A patient with Hb 7 was to be transfused with 2 packs of blood. The first pack was
transfused in 2 hours after which vitals were stable and then the transfusion of the next
pack was started . But suddenly the patient develops breathlessness and hypertension.
What can be the cause of this sudden reaction?

A. transfusion related circulatory overload (TACO)


B. allergic reaction to transfused blood
C. transfusion related acute lung injury (TRALI)
D. transfusion related acute renal failure

Answer- C. transfusion related acute lung injury (TRALI)

Transfusion-related acute lung injury (TRALI) is a serious blood transfusion complication characterized by

ed co
the acute onset of non-cardiogenic pulmonary edema following transfusion of blood products.

Apps
Hypotension, and fever that develop within 6 hours after transfusion and usually resolve with supportive
care within 48 to 96 hours. Although hypotension is considered one of the important signs in diagnosing
TRALI, hypertension can occur in some cases.

A Little Help to Get Started


132.The severity of mitral stenosis can be judged by-

A. Intensity of murmur
B. Duration of murmur
C. Left ventricular S3
D. Loud S1

Answer- B. Duration of murmur

Duration depends on severity of MS.


In severe MS, the mid diastolic murmur is long and merges with the presystolic murmur to produce
holodiastolicmurmur..

For All Previous Year AIIMS PG Questions Visit

www.medicoapps.org/aiimspg/
Surgery

133. Which of the following does not cause head and neck squamous cell carcinoma

A. Alcohol
B. Betel nut
C. HPV
D. EBV

Answer- D .EBV

The most important risk factors for developing HNSCC are tobacco smoking and alcohol consumption.
Consumed in betel quids containing areca nut increases the risk of developing HNSCC.
HNSCCs of the oral cavity and oropharynx, are becoming more prevalent, which may be related to an
increase in oral and oropharyngeal HPV infection.

134.Most common

A. Osteoporosis
B. Paget's dis
AC. Osteopetrosis
ed co
cause of intertrochanteric fracture in old is

Little Help to Get Started


Apps

D. Osteomalacia

Answer- A. Osteoporosis

 Having osteoporosis
 Having a history of other bone problems or fractures
 Having low bone density and low muscle mass

135. Numbness on shaving after parotidectomy due to which nerve injury?

A. Facial nerve
B. Auriculotemporal nerve
C. Greater auricular nerve

For All Previous Year AIIMS PG Questions Visit

www.medicoapps.org/aiimspg/
D. Occipital nerve

Answer- B. Auriculotemporal nerve

136. Most common functional tumors of endocrine pancreas?

A. Vipoma
B. Gastrinoma
C. Glucagonoma
D. Somatostatinoma

Answer- B. Gastrinoma

Insulinoma is the most common functional tumour but if that wasnt there in the optios then gastrinoma is
the next answer.

ed co
guarding in upper epigastrium. Chest X-ray normal. What to do next?

A. UGI endoscopy
AB. Serum
L i lipase
ttle Help to Get Started
Apps
Chronic alcoholic comes with pain epigastrium and recurrent vomiting. On examination
137.

C. CECT
D. Alcohol breath test

Answer- B. Serum lipase

Serum amylase and lipases are the initial investigations done in patients with acute pancreatitis.
CECT is the investigation of choice but the initial investigation in such patients.

138.During diagnostic laparoscopy for undescended testis, there are absent testicular vessels.
What should be done next?

A. Explore further
B. Nothing is to be done

For All Previous Year AIIMS PG Questions Visit

www.medicoapps.org/aiimspg/
C. Inguinal exploration
D. Scrotal examination

Answer- B. Nothing is to be done

Nothing is to be done. Absent testicular vessels denotes that there has been intra- uterine torsion and in
that case further exploration is required.

139. Which of the following structure is not removed in radical neck dissection-

A. Sublingual
B. Submandibular
C. Tail of parotid
D. Level 2 b lymph nodes

Answer- A. Sublingual

ed co
Structures removed during radical neck dissection-
 Level 1-5 In
 Sternocleidomastoid, IJV, spinal accessory nerve
 Submandibular gland
Apps
 Tail of parotid
A Little Help to Get Started
140.All are features of SIRS except-

A. RR> 24 & Paco2 <22mm hg


B. WBC >11 or <4
C. Temperature <36 and >38
D. PR >90

Answer- A. RR> 24 & Paco2 <22mm hg

For All Previous Year AIIMS PG Questions Visit

www.medicoapps.org/aiimspg/
141. Treatment of choice in a patient with a staghorn calculus with mild hydronephrosis?

A. ESWL
B. PCNL
C. RIRS
D. Open surgery

Answer- B. PCNL
PCNL is the best answer as the patient has hydronephrosis and in an obstructed system ESWL is not useful
as stone fragments don't get cleared.

142.A patient presents to the ER after a RTA with multiple rib injuries. He is conscious, speaking
single words.
RR= 40/ minute, BP= 90/40mm Hg. What is the next immediate step in management?

A. Intubate the patient


B. Urgent fluid infusion
C. Chest X- ray
D. Needle insertion in 2nd ICS

Answer- D. Needle insertion in 2nd ICS

For All Previous Year AIIMS PG Questions Visit

www.medicoapps.org/aiimspg/
This is a case of pneumothorax. Although the new ATLS update is 5th intercostal space in mid axillary line
but in this question we will go with a time tested method of needle in the 2nd ICS.

143. Percentage of SSI rate in patients with a clean contaminated wound?

A. 1-2%
B. <10%
C. 10- 20 %
D. 20- 30 %

Answer- B. <10%

 Clean wound- 1- 2%
 Clean contaminated- 7- 10%
 Contaminated- 10- 20%
 Dirty- >20%

144.Which one of the following does not classify as locally advanced breast cancer?

A. Tumour more than 4cm


B. Inflammatory breast cancer
C. Chest wall involvement
D. Skin involvement

Answer- A. Tumour more than 4cm

For All Previous Year AIIMS PG Questions Visit

www.medicoapps.org/aiimspg/
Locally advanced breast cancer-
T4 N2 N3 M0

145. Which of the following is not a component of quick SOFA (qSOFA) scoring?

A. Bilateral undilated pupils


B. Altered Mentation
C. Glasgow Coma Score
D. SBP <= 100 mm Hg

Answer- A. Bilateral undilated pupils

Assessment qSOFA score

Low blood pressure (SBP ≤ 100 mmHg) 1

High respiratory rate (≥ 22 breaths/min) 1

ed co
Altered mentation (GCS ≤ 14) 1

Apps
146. Which of the following would you consider orthostatic hypotension?

AA. SBP
L fall
i tby t20 lmmehg, DBPHfalleby 10l p t o 3 minutes
mm hg within Get Started
B. SBP fall by 20 mm hg, DBP fall by 10 mm Hg within 6 minutes
C. SBP fall by 30 mm hg, DBP fall by 20 mm Hg within 3 minutes
D. SBP fall by 30 mm hg, DBP fall by 20 mm hg within 6 minutes

Answer- A. SBP fall by 20 mm hg, DBP fall by 10 mm hg within 3 minutes

Orthostatic hypotension is defined as a decrease in systolic blood pressure of 20 mm Hg or a decrease in


diastolic blood pressure of 10 mm Hg within three minutes of standing when compared with blood
pressure from the sitting or supine position.

147. Identify the image of the instrument-

For All Previous Year AIIMS PG Questions Visit

www.medicoapps.org/aiimspg/
A. Bent Hohmann Retractors
B. Bone Holding and Reduction Clamps and Forceps
C. Bone cutter
D. Bone nibbler

Answer- D. Bone nibbler

The GPC straight & curved bone nibbler (double action) is used for nibbling the bone.
The straight nibbler is used for general use and the curved nibbler is used for spinal surgery.

148.Device shown in the image is used for what purpose?

A. Prevent viral infection


B. Prevent transfusion related reactions
C. Prevent infection
D. None
Answer- B. Prevent transfusion related reactions
It is a leuko- reduction filter which prevents transfusion related reactions

149. What is the best way to manage the wound given in the image-

A. Cleaning & dressing

For All Previous Year AIIMS PG Questions Visit

www.medicoapps.org/aiimspg/
B. Cleaning & debridement
C. Use allograft
D. Use autograft
Answer- A. Cleaning & dressing

150.Which is not associated with GDM (Gestational DM)?

A. Past history of fetal abnormality


B. Obesity
C. Hypertension
D. Macrosomia

Answer- C. Hypertension

A previous diagnosis of gestational diabetes or prediabetes, impaired glucose tolerance, or impaired

ed co
fasting glycaemia.
Being overweight, obese
A previous pregnancy which resulted in a child with a macrosomia.
Apps
A Little Help to Get Started
151. False statement about Thyroglossal Duct cyst is-

A. Infected thyroglossal cyst from sinus


B. Lined by pseudostratified columnar epithelium
C. 40% cases have sub hyoid location
D. It is due to congenital

Answer- A. Infected thyroglossal cyst from sinus

Thyroglossal cysts are the most common cause of midline neck masses and are generally located caudal to
(below) the hyoid bone.
Thyroglossal Duct Cysts are a birth defect.
A thyroglossal cyst is lined by pseudostratified, ciliated columnar epithelium while a thyroglossal fistula is lined by
columnar epithelium.
A thyroglossal duct cyst may rupture unexpectedly, resulting in a draining sinus known as a thyroglossal fistula.

For All Previous Year AIIMS PG Questions Visit

www.medicoapps.org/aiimspg/
152. Identify the disorder (undersurface of breast) as shown in the photograph below ?

A. Mondor's disease
B. Paget's disease
C. Mastitis
D. Chronic inflammatory abscess

Answer- A. Mondor's disease

The condition shown in the picture above represents Mondor's disease.


Mondor's disease (also known as "Mondor's syndrome of superficial thrombophlebitis") is a rare condition
which involves thrombophlebitis of the superficial veins of the breast and anterior chest wall. It sometimes
occurs in the arm or penis. In axilla, this condition is known as axillary web syndrome.

Obs / Gyne

153. Arrange the following steps in sequence as it occurs in spermatogenesis.

A. Spermatocyte
B. Spermatogonia
C. Spermatozoa
D. Spermatid

For All Previous Year AIIMS PG Questions Visit

www.medicoapps.org/aiimspg/
Ans-b)Spermatogonium> a)Spermatocyte >d)Spermatid >c)Spermatozoa

154. Chicken pox infection in mother most dangerous for the baby when it occurs at

A. 14-28 weeks
B. 3 -9 weeks
C. 28-37weeks
D. 8-14weeks

Ans. C. 28-37weeks

 Before 28 weeks pregnant: there's no evidence you are at increased risk of suffering a miscarriage.
However, there's a small risk baby could develop foetal varicella syndrome (FVS). FVS can damage
the baby's skin, eyes, legs, arms, brain, bladder or bowel.
 Between weeks 28 and 36 of pregnancy: the virus stays in the baby's body but doesn't cause any
symptoms. However, it may become active again in the first few years of the baby's life, causing
shingles.
 After 36 weeks of pregnancy: baby may be infected and could be born with chickenpox.

155.All are used in PPH expert :

A. Mife pristine
B. Carboprost
C. Misoprostol
D. Ergometrine

For All Previous Year AIIMS PG Questions Visit

www.medicoapps.org/aiimspg/
Ans. A. Mifepristone

156.Which of the following can be used to delay preterm contractions of uterus with best
neurological outcome for the fetus?

A. Mgso4
B. Nifidipine
C. Ritodrine
D. Isoxprine

Ans. A.MgSo4
Antenatal magnesium sulfate for both tocolysis and fetal neuroprotection in premature rupture of the
membranes before 32 weeks' gestation.

ed co
Other tocolytic drugs:


labour and delivery. Apps
Ritodrine, salbutamol and magnesium sulphate are tocolytic drugs used to terminate preterm

Other tocolytic drugs are isoxsuprine, indomethacin, calcium channel blockers, glyceryl trinitrate,
atosiban and glyceryl trinitrate.
A Little Help to Get Started

157.Assertion reason type q:


A: Hot flushes are experienced by women during menopause
R:Hot flushes are due to withdrawal or fluctuation of estrogen.

A. Both assertion and reason are true and reason is the correct explanation of assertion
B. Both assertion and reason are true but reason is not the correct explanation of assertion
C. Assertion is true but reason is false
D. Reason in true but assertion is false

Ans.A.Both assertion and reason are true and reason is the correct explanation of assertion

 Hot Flushes :They are the 'hallmark' of menopause. Hot flushes are described as recurrent transient
period of flushing, sweating and a sensation of heat often accompanied by palpitations, feelings of
anxiety and sometimes followed by chills.

For All Previous Year AIIMS PG Questions Visit

www.medicoapps.org/aiimspg/
 The entire episode lasts no more than 1-3 minutes and recurs 5-10 times / day (can occur upto 30
times a day).
 Short term estrogen therapy results in resolution of hot flushes.

158. Best Age to start bone mineral density test in female.

A. After 50 years
B. After 55 years
C. After 60 years
D. After 65 years

Ans.d.After 65 years
NOF recommends:
Bone density test :


ed co
In woman age 65 or older.
In man age 70 or older.

A Little Help to Get Started


Apps

159.A 25 year old nulliparous woman at 35 weeks' gestation comes to the labor and delivery
ward complaining of contractions, a headache, and flashes of light in front of her eyes. Her
pregnancy has been uncomplicated except for an episode of first trimester bleeding that
completely resolved. She has no medical problems.

Her temperature is 37 C (98.6 F), blood pressure is 160/110 mm Hg, pulse is 88/minute, and
respirations are 12/minute. Examination shows that her cervix is 2 centimeters dilated and 75%
effaced, and that she is contracting every 2 minutes. The fetal heart tracing is in the 140s and
reactive.

Urinalysis shows 3+ proteinuria. Laboratory values are as follows: leukocytes 9,400/mm3,


hematocrit 35%, platelets 101,000/mm3. Aspartate aminotransferase (AST) is 200 U/L, and ALT
300 U/L. Which of the following is the most appropriate next step in management?

A. Administer oxytocin
B. Discharge the patient
For All Previous Year AIIMS PG Questions Visit

www.medicoapps.org/aiimspg/
C. Encourage ambulation
D. Start magnesium sulphate

Ans. D. Start magnesium sulphate

 This patient has severe preeclampsia. Preeclampsia is diagnosed on the basis of hypertension,
edema, and proteinuria.

 Severe preeclampsia may be diagnosed when the patient has one of the following: a headache that
does not respond to analgesics, visual changes, seizure, very elevated blood pressures, pulmonary
edema, elevated liver function tests, severe proteinuria, oliguria, an elevated creatinine,
thrombocytopenia, hemolysis, intrauterine growth restriction, or oligohydramnios.

 The management of severe preeclampsia after 32 weeks is with delivery.

 Prior to 32 weeks, consideration may be given to expectant management of the patient depending
on the clinical circumstances. This patient is at 35 weeks' with headache, visual changes, elevated
blood pressures, thrombocytopenia, and elevated liver function tests. She, therefore, needs to be
delivered.

She appears to already be in labor as she is contracting every two minutes and her cervix is dilated

ed co

and effaced.

 At this point, magnesium sulfate should be started. Apps


 Magnesium sulfate has consistently been demonstrated to be the most effective medication for
A Little Help to Get Started
seizure prophylaxis in women with preeclampsia.

 To administer oxytocin would not be necessary for this patient. She appears to already be in labor
with contractions every two minutes.

 To discharge the patient would absolutely be incorrect. Severe preeclampsia need to be delivered
or, at the very least, admitted to the hospital. There is no role for discharging a patient home in the
management of severe preeclampsia.

 To encourage ambulation would also be incorrect. Severe preeclampsia should be kept on bed rest.

Ref: Cunningham F.G., Leveno K.J., Bloom S.L., Hauth J.C., Rouse D.J., Spong C.Y. (2010). Chapter 34.
Pregnancy Hypertension. In F.G. Cunningham, K.J. Leveno, S.L. Bloom, J.C. Hauth, D.J. Rouse, C.Y. Spong
(Eds), Williams Obstetrics, 23e.

For All Previous Year AIIMS PG Questions Visit

www.medicoapps.org/aiimspg/
160.P3L3 came to opd with postcoital bleeding via and pap positive p/v cervix hypertrophied
bleed on touch diagnosis

A. CA cervix
B. Fibroid
C. Cervicitis
D. Cervical polyp

Ans. A. CA cervix

SYMPTOMS:
 Arises from: Squamo-columnar junction
 Earliest symptom: Post-coital bleeding
As the cancer progresses, symptoms may include:
 Unusual vaginal discharge
 Vaginal bleeding between periods
 Bleeding after menopause
 Pyometra

ed co
 Bleeding or pain during sex



MC site: Ectocervix
Lymph nodes affected: Obturator, hypogastric and external iliac
Time taken for conversion of CIN to invasive Ca: 10 years
Apps
 MC type: Squamous cell Ca
A Little Help to Get Started


100% cure rates are seen in: CIS
Uremia: altered sensorium and is having hiccups
 MC cause of death: Renal failure
 Ca cervix can be prevented by screening

Both positive Pap smear and VIA test is suggestive of CA cervix

161.A 35 year old female presented with post coital bleeding. Next step of management is:

A. Pap smear and colposcopy


B. Visual inspection with acetowhite
C. Visual inspection with lugol's iodine
D. Colposcopy directed biopsy

Ans. A. Pap smear and colposcopy

For All Previous Year AIIMS PG Questions Visit

www.medicoapps.org/aiimspg/
 Postcoital bleeding (PCB) has many causes, including vaginitis, cervicitis, cervical dysplasia and
malignancy, uterine lesions, and pregnancy.
 Some women with postcoital bleeding may have pathologic lesions identified at colposcopic
evaluation that had been missed by Pap smear screening.
 Thus, colposcopic examination is considered for women with unexplained postcoital bleeding.

Ref: Hoffman B.L., Schorge J.O., Schaffer J.I., Halvorson L.M., Bradshaw K.D., Cunningham F.G., Calver L.E.
(2012). Chapter 8. Abnormal Uterine Bleeding. In B.L. Hoffman, J.O. Schorge, J.I. Schaffer, L.M. Halvorson,
K.D. Bradshaw, F.G. Cunningham, L.E. Calver (Eds), Williams Gynecology, 2e.

162. 28 yr with infertility lapro tube uterus healthy ovary diagnosis

A. PCOS
B. Ovarian cyst
C. Fibroid
D. Endometriosis

ed co
Ans.D. Endometriosis

 In endometriosis, cause of infertility is


Apps
 Immobility of tubes
 Anovulation
A Little Help to Get Started


Tubal block
Male factor: 30%
 Tubal, uterine & peritoneal factor: 25%
 Ovarian factor: 25%
 Cervical factor: 10%
 Unexplained factor: 10%

163. Fetal ECHO shows congenital heart block, what should be the mother screened for?

A. SLE
B. Myxoma
C. APLA
D. None

Ans. a). SLE


For All Previous Year AIIMS PG Questions Visit

www.medicoapps.org/aiimspg/
Congenital heart block is a rare disorder that occurs in about one out of 22,000 live births.
In most cases, the cause is not known, but babies of mothers with lupus or other autoimmune diseases, or
babies with congenital heart disorders, are at higher risk.
Pregnant women who have autoimmune diseases, such as lupus or Sjogren's syndrome, are at an
increased risk of having a baby with congenital heart block.
Also, a tumor on the baby's heart can cause heart block.

164.A 49 year old female was prescribed hormone replacement therapy (HRT). HRT is useful in all
of the following, EXCEPT:

A. Flushing
B. Osteoporosis
C. Vaginal atrophy
D. Coronary heart disease

Ans . D: Coronary heart disease

Definite Benefits

A Symptoms
Osteoporosis
ed co
Benefits and Risks of Postmenopausal Hormone Therapy (HT):

L i t tof lmenopause
e H(e.g,eFlushing,
l p vaginal
to atrophy)
Get Started
Apps

Definite Risks

 Endometrial cancer
 Venous thromboembolism
 Breast cancer
 Gallbladder disease

Probable or Uncertain Risks and Benefits

 Coronary heart disease


 Stroke
 Ovarian cancer
 Colorectal cancer
 Diabetes mellitus
 Cognitive dysfunction

For All Previous Year AIIMS PG Questions Visit

www.medicoapps.org/aiimspg/
Ref: Manson J.E., Bassuk S.S. (2012). Chapter 348. The Menopause Transition and Postmenopausal
Hormone Therapy. In Longo D.L., Fauci A.S., Kasper D.L., Hauser S.L., Jameson J, Loscalzo J (Eds), Harrison's
Principles of Internal Medicine, 18e.

Ortho

165. Match the following-

A. Pilons fracture- Trimalleolar #


B. Chauffeur fracture-fracture of the distal part of the tibia
C. Cotton fracture- fracture of radial styloid process
D. Runner fracture-Lateral malleolar #

Answer- A: A-2, b-3, c-4, d-1

 Pilons - fracture of the distal part of the tibia


 Chauffeur- fracture of radial styloid process.
 cotton- Lateral malleolar #
 Runners fracture- Trimalleolar #

166.Image showing x-ray of osteolytic lesion on lateral condyle of femur. What will be the line of
treatment?

A. extended curettage with bone autograft


For All Previous Year AIIMS PG Questions Visit

www.medicoapps.org/aiimspg/
B. extended curettage with bone allograft
C. curettage only
D. Bone biopsy
Answer- D. Bone biopsy

167. Specific for Ankylosing spondylitis is true?

A. HLA
B. B27
C. Sacroileitis
D. Raised ESR

Answer- C. Sacroileitis

168.
to avascular necrosis?

A A. LProximal
ed co
ittle Help to Get Started
B. Distal
Apps
Scaphoid fracture at waist with retrograde blood supply. Which segment is most susceptible

C. Middle
D. Scaphoid tubercle

Answer- A. Proximal
 Most common site of scaphoid fracture is Waist.
 Fractures can occur essentially anywhere along the scaphoid, but distribution is not even:
A. waist of scaphoid: 70-80%
B. proximal pole: 20%
C. distal pole (or so-called scaphoid tubercle): 10%

For All Previous Year AIIMS PG Questions Visit

www.medicoapps.org/aiimspg/
169. Which of the following is true about anterior shoulder dislocation -

A. It is the most common type of shoulder dislocation


B. It is most commonly subclavicular
C. Patient keeps his arm in saluting position
D. Injury to brachial plexus may occur

Answer- A. It is the most common type of shoulder dislocation

Most common type of shoulder dislocation is anterior dislocation (subcoracoid being most common).
Patient keeps his arm by the side of the body in a position of abduction and internal rotation.

Pediatrics

For All Previous Year AIIMS PG Questions Visit

www.medicoapps.org/aiimspg/
170.A child can walk upstairs one step at a time, can ride cycle but can't jump can also speak
sentences, can tell his/her name gender but finds difficult to narrate the story.What is her actual
developmental age

A. 1 yrs
B. 2
C. 3
D. 4

Ans. b) 2years

GROSS MOTOR MILESTONES:

Age Milestone
3 months Neck holding
5 months Rolls over
6months Sitting supported
8months Sitting without support

ed co
9months Stands with support
12months
15months
Stands without support, Walks but falls
Walks alone, Creeps upstairs
Apps
18months Runs, explores drawers
A
2 years L i t t lWalks
e upstairs
H (baby
e l steps),
p Jumpsto Get Started
3 years Walks upstairs (alternate feet), rides tricycle
4 years Hops on one foot, walks downstairs (alternate feet)

171.Video based question


AIIMS NICU - neonate shown on O2 with nasal prongs, not intubated, sister shown inserting a tube
through the mouth, camera zooms in, 18 (calibration) mark seen, the other (green) end as of now not
connected to anything, video ends, what is the procedure that is being done?
VIDEO LINK: https://media.giphy.com/media/ln0aVCWqPvUtTWqBtT/giphy.gif

A. Oral suction
B. Oropharyngeal suction
C. Nasogastric tube insert
D. Orogastric tube insertion

For All Previous Year AIIMS PG Questions Visit

www.medicoapps.org/aiimspg/
Ans.Orogastric tube insertion

Pediatric patientpresented with 45 mins h/o continuous convulsions.


172.
CASE 1: SR told to give iv lorazepam but J.R cant secured iv line. Then what he has given?

A. Rectal diazepam
B. Inhalation Phenobarbital
C. IV carbamazepine
D. Subcutaneous midazolam

Answer A. Rectal diazepam


 Benzodiazepines are Drug of Choice and in pediatricpatients rectal route should be preffered.

Reference - https://www.uspharmacist.com/article/emergent-treatment-of-status-epilepticus-in-children

CASE -2 SR visit again but the condition is not improved but this time IV cannula was set. What

ed co
drug should be given now?

A. Midazolam Apps
B. I/V Phenobarbital

AC. Oral
L ivalproate
ttle Help to Get Started
D. IV carbamazepine

Ans. In this case the First Line has failed .So for second line therapy I/V Phenobarbital is preffered
Reference - https://www.uspharmacist.com/article/emergent-treatment-of-status-epilepticus-in-children

173. Sequential arrangement of fetal scans

A. Growth scan
B. Triple marker
C. Anomalous Scan
D. NT scan

Ans: d,b,c,a.....
For All Previous Year AIIMS PG Questions Visit

www.medicoapps.org/aiimspg/
The NT scan must be done between 11 and 14 weeks pregnant, because this is when the base of baby's
neck is still transparent. (The last day for scan is 13weeks and 6 days pregnant.)
Triple marker test is performed in pregnant women at the end of first trimester and the beginning of the
second trimester.
The anomaly scan, also sometimes called the anatomy scan, 20 week ultrasound, or level 2 ultrasound, is a
pregnancy ultrasound performed between 18–22 weeks
A growth scan is an ultrasound scan that determines whether your baby's growth is normal. Doctors
typically recommend it for women during the third trimester of pregnancy; one of the reasons it is also a
fetalgrowth scan between 28 weeks and 32 weeks of pregnancy.

174.4yr boy absence of right testes, diagnostic laparoscopy done, a blind end vessel seen what to
do next?

A. Open laprotomy
B. Nothing to be done
C. Scrotal approach


ed co
D. Inguinal approach

Ans. D. Inguinal approach

A L i t t l e H e l p t o G e t S t a r t e d

Cryptorchidism means hidden testis.
Apps
Cryptorchidism is the most common congenital abnormality of the genitourinary tract.

An absent testis may be due to agenesis or atrophy secondary to intrauterine vascular compromise
also known as the "vanishing testis syndrome".
 Bilaterally absent testes is anorchia which is 10% cases.
 More common on Right Side.
 Complications of undescended testes
 Torsion can be seen in incomplete testicular descent
 Sterility is seen in bilateral cases (especially intra-abdominal testes)
 Incomplete testicular descent predisposes to malignant disease; cancer is more common in an
incompletely descended testes-orchidopexy may or may not diminish the risk.
 Atrophy of an inguinal testes before puberty may possibly be caused by recurrent minor trauma.
 In patients with a unilateral nonpalpable testis, a descended testis that is larger than expected
suggests an atrophic undescended testis; confirmation requires surgical intervention typically via
diagnostic laparoscopy to seek an intra-abdominal testis or confirm testicular agenesis. However,
scrotal or inguinal exploration is sometimes done if a testicular remnant distal to the internal
inguinal ring is suspected.

For All Previous Year AIIMS PG Questions Visit

www.medicoapps.org/aiimspg/
A 4 year-old boy, develops a large erythematous rash around the site of a mosquito bite.
175.
One month later, he is taken to a pediatrician because of a puffy face and swollen ankles.
The scanty urine sample has a reddish-brown hue, and contains both red blood cells and protein.
Which of the following distinctive features would be most likely to be seen on renal biopsy?

A. Fusion of podocyte foot processes


B. IgA in the mesangium
C. Linear IgG deposits
D. Subepithelial electron dense humps

Ans. D. Subepithelial electron dense humps

The disease is poststreptococcal glomerulonephritis, which can follow either streptococcal pharyngitis
(one to two weeks after the infection) or skin infection (three to six weeks after the infection).

ed co Apps
The child is showing signs of both nephritis (hematuria) and nephrosis (puffy face and swollen ankles).

The characteristic feature of this disease on renal biopsy is the presence of subepithelial humps, visible
either by light or electron microscopy.

A Little Help to Get Started


Fusion of podocyte foot processes suggests minimal change disease.

IgA in the mesangium suggests Berger's disease.

Linear IgG deposits suggests anti-glomerular basement membrane disease, which is called Goodpasture's
disease if it is accompanied by pulmonary damage.

Onion-skinning of renal arterioles suggests malignant hypertension.

176.A boy presented with a history of fever, headache and vomiting since 3 days and
disorientation since I day. On examination, neck rigidity was positive. He also had 1 episode
of generalized tonic clonic seizures (GTC) and then he became unconscious. CECT was found
to be normal. CSF examination reveals a cell count of 300 cells/mm3 (polymorphs: 50-70%),

For All Previous Year AIIMS PG Questions Visit

www.medicoapps.org/aiimspg/
protein 70 mg/dL, sugar 50 mg/dL (blood: 95 mg/dL). Which of the following is the most
likely diagnosis?
A. Pyogenic meningitis
B. Tubercular meningitis
C. Herpes encephalitis
D. Cerebral malaria

Ans. a. Pyogenic meningitis (ReJ: Harrison l9/e p887, 891, llll, l8/e p1029, 1692, 3414)

The classic CSF abnormalities in bacterial meningitis are:

 Elevated opening pressure (> 180 mm H2O in 90%)


 Polymorphonuclear leakocytosis (>100 cells/ul in 90%)
 Decreased glucose concentration (
 Elevated protein concentration (>45 mg/dl)

177. Arrange the following in the sequence of closure :


A. Umbilical Artery
B. Umbilical Vein
C. Ductus venosus
D. Foramen ovale
E. Ductus arteriosus

Ans.E>D>C>A>B

For All Previous Year AIIMS PG Questions Visit

www.medicoapps.org/aiimspg/
Within a week of birth, the neonate's umbilical vein is completely obliterated and is replaced by a fibrous
cord called the round ligament of the liver (also called ligamentum teres hepatis).
Closure of the umbilical vein usually occurs after the umbilical arteries have closed

Skin
After sequential arrangement of blistering disorder starting from superficial to deep blisters,
178.
which is the deepest among these?

B. EBD ed co
A. IgA pemphigus

AC. PVL i t t l e H e l p t o G e t S t a r t e d
D. Bullous pemphigoid+
Apps

Ans. C. PV

 Pemphigus vulgaris (PV) is a rare and serious (potentially life-threatening) condition that causes
painful blisters to develop on the skin and lining of the mouth, nose, throat and genitals.
 The blisters are fragile and can easily burst open, leaving areas of raw unhealed skin that are very
painful and can put you at risk of infection.
 H&E: Subepidermal bullae

For All Previous Year AIIMS PG Questions Visit

www.medicoapps.org/aiimspg/
 DIF: linear IgG and C3 deposition at the basement membrane zone

179. Identify the condition below?

A. Tardieu spots
B. ITP
C. Vasculitis
D. DIC spots

Ans. A. Tardieu spots


 Asphyxial petechial haemorrhages do occur on the serosal surfaces of internal organs eg.heart and
lung as shown in the pictures.
 They were first described by Tardieu in the 19th century and were named after him as tardieu
spots.
 It were considered at that time and for a long period after as pathognomonic sign of asphyxial
deaths.
 Such petechial haemorrhages do occur in deaths related toinfections,blood coagulopathy disorders
and micro emboli.
 It can also occur due to rupture of engorged capillaries in the dependent parts.
 It can also be noticed in some cases of natural deaths, so it is by no means are pathognomonic of
asphyxial deaths.

180.Identify the condition below and which of the following is its most characteristic microscopic
finding?

A. Intracellular inclusion bodies


B. Multinucleated cells

For All Previous Year AIIMS PG Questions Visit

www.medicoapps.org/aiimspg/
C. Subepithelial Bullae
D. Councilman body

Ans. A. Intracellular inclusion bodies

(Ref: Robbins 9/e p1176; Fitzpatrick 6/e p2347)

 All findings are highly suggestive of Molluscum contagiosum, which is caused by the poxvirus.

Molluscum contagiosum:

 On microscopic examination, lesions show cuplike verrucous epidermal hyperplasia. The


diagnostically specific structure is the molluscum body, which occurs as a large, ellipsoid,
homogeneous, cytoplasmic inclusion in cells of the stratum granulosum and the stratum corneum.
 In H&E stain, these inclusions are eosinophilic in the blue-purple stratum granulosum and acquire a
pale blue hue in the red stratum corneum.
 Numerous virions are present within molluscum bodies.

Image shown Child having vesicular lesions on lower lip, palms & soles, question was asked
181.
which of the following group causes the disease?

A. HSV – 1
B. Picornaviridae

For All Previous Year AIIMS PG Questions Visit

www.medicoapps.org/aiimspg/
C. Togaviridae
D. Pox virus

Ans. C. Togaviridae
 Vesiculobullous lesions were common cutaneous manifestation of chikungunya fever in infants.
 Pigmentary lesions were generalized, brownish black and predominantly involved face and
extremities.
 Generalized maculopapular eruption or erythema with islands of normal skin were noted, mostly 2-
3 days after onset of fever.
 It started on trunk and then spread centrifugally involving face, palms and soles.
 Mucosal regions were spared.

 Mostly it subsided in 4-5 days, but in 5 cases there was peeling of the skin over body, palms and
soles, resembling staphylococcal scalded skin syndrome (SSSS).

182. Which of the following cells not present in stratum basale layer?

A. Langerhans cells
B. Merkel cells
C. Keratocyte
D. Melanocytes

Ans. A. Langerhans cells


 The stratum basale is primarily made up of basal keratinocyte stem cells, which can be considered
the stem cells of the epidermis.
 They divide to form the keratinocytes of the stratum spinosum, which migrate superficially.
 Other types of cells found within the stratum basale are melanocytes (pigment-producing cells),
and Merkel cells (touch receptors).

Anaesthesia
For All Previous Year AIIMS PG Questions Visit

www.medicoapps.org/aiimspg/
183.Action of proparacaine starts within?

A. 2 minutes
B. 5 minutes
C. 15 minutes
D. 20 minutes

Ans. A. 2 minutes
 Proparacaine and tetracaine are indicated to produce local anesthesia of short duration for
ophthalmic procedures including measurement of intraocular pressure (tonometry), removal of
foreign bodies and sutures, and conjunctival and corneal scraping in diagnosis and gonioscopy.
 Onset of action:
o Proparacaine—Within 20 seconds.
o Tetracaine—Approximately 15 seconds.
 Duration of action:
o Proparacaine—15 minutes or longer.
o Tetracaine—10 to 20 minutes; average 15 minutes.

ed co Psychiatry

184.Which one of the following disorders is related to sense of unreality?


A Little Help to Get Started
Apps
A. Depersonalization disorder
B. Derealization disorder
C. Delusion
D. Phobias

Ans. A. Depersonalization disorder

Depersonalization:

 Depersonalization is a sense of unreality or strangeness concerning the self, manifested by feeling


detached from and being an outside observer of one's mental processes or body.
o A patient who experiences depersonalization may describe feeling like things “I have
stepped outside of myself and am watching myself doing things”.
Derealization:
 Derealization refers to feeling detached from one's environment so that the sense of the reality of
the external world is lost.

For All Previous Year AIIMS PG Questions Visit

www.medicoapps.org/aiimspg/
o E.g, I feel as if everything is unreal and those around me are actors in a play or two
dimensional cardboard figures.

185. Which of the following is not included in the grades of insight?

A. Intellectual
B. Emotional
C. Psychological
D. Judgemental

Ans. D. Judgemental
 Neziroglu and Stevens Proposed four different levels of insights:
o True emotional insight.
o Intellectual insight

ed co
o Partial internally and externally based insight



o Denial Of Illness.
Apps
True emotional insight Is representative of the highest level of insight possible.
In it the patients' awareness and understanding Of their own thoughts, feelings and motives can be
used to change behavior.

A Little Help to Get Started

186.Which of the following dementia is associated with visual hallucinations?

A. Lewy body dementia


B. AIDS related Dementia
C. Huntington's disease
D. Mixed dementia

Ans. A. Lewy body dementia –

 Lewy body dementia, also known as dementiawithLewy bodies, is the second most common type
of progressive dementia after Alzheimer's disease dementia.
 Protein deposits, called Lewy bodies, develop in nerve cells in the brain regions involved in
thinking, memory and movement (motor control).

For All Previous Year AIIMS PG Questions Visit

www.medicoapps.org/aiimspg/
ed co
A Little Help to Get Started
Apps

For All Previous Year AIIMS PG Questions Visit

www.medicoapps.org/aiimspg/

Potrebbero piacerti anche